ONLINE ORTHOPEDIC MCQS ONCOLOGY/TUMOR08
ONLINE ORTHOPEDIC MCQS ONCOLOGY/TUMOR08
1. A 13-year-old girl has had increasing left hip pain for the past 4 months. A radiograph, bone scan, MRI scan, and photomicrograph are shown in Figures 1a through 1d. Which of the following immunohistochemistry results would confirm the most likely diagnosis?
1- Cytokeratin positive
2- PAS negative
3- Reticulin positive
4- MIC-2 positive
5- Vimentin negative
PREFERRED RESPONSE: 4
DISCUSSION: The imaging studies show a permeative lesion of the left hemipelvis with a large soft-tissue mass. The photomicrograph demonstrates a small blue cell tumor with pseudorosettes. The most likely diagnosis is primitive neuroectodermal tumor (Ewing’s sarcoma family of tumors). MIC-2 is a highly sensitive and specific marker for this family of tumors. Cytokeratin is an epithelial marker. Vimentin is a mesenchymal marker. Thus, Ewing’s sarcomas are cytokeratin negative and vimentin positive. Before discovery of the MIC-2 antigen, PAS and reticulin stains were commonly used to help differentiate Ewing’s sarcoma from lymphoma. In contrast to lymphoma, Ewing’s sarcomas are typically PAS positive and reticulin negative.
REFERENCES: Halliday BE, Slagel DD, Elsheikh TE, et al: Diagnostic utility of MIC-2 immunocytochemical staining in the differential diagnosis of small blue cell tumors. Diagn Cytopathol 1998;19:410-416.
Llombart-Bosch A, Navarro S: Immunohistochemical detection of EWS and FLI-1 proteins is Ewing sarcoma and primitive neuroectodermal tumors: Comparative analysis with CD99
(MIC-2) expression. Appl Immunohistochem Mol Morphol 2001;9:255-260.
2. Which of the following is the preferred treatment for symptomatic localized pigmented villonodular synovitis (PVNS) of the knee?
1- Observation
2- External beam radiation therapy
3- Intra-articular radiation therapy
4- Resection of nodule only
5- Open complete synovectomy
PREFERRED RESPONSE: 4
DISCUSSION: Localized PVNS is a variant of the disease process where the synovial proliferation occurs in one area and usually presents as a discrete mass. It has been effectively treated with complete excision. This may be performed arthroscopically or with arthrotomy. Complete synovectomy and radiation therapy are unnecessary to eradicate the localized form of PVNS.
REFERENCES: Tyler WK, Vidal AF, Williams RJ, et al: Pigmented villonodular synovitis.
J Am Acad Orthop Surg 2006;14:376-385.
Kim SJ, Shin SJ, Choi NH, et al: Arthroscopic treatment for localized pigmented villonodular synovitis of the knee. Clin Orthop Relat Res 2000;379:224-230.
3. A previously asymptomatic 14-year-old girl sustained a twisting injury to her ankle. Radiographs are shown in Figures 2a and 2b. Management should consist of
1- observation.
2- MRI.
3- bone scan.
4- biopsy.
5- curettage and bone grafting.
PREFERRED RESPONSE: 1
DISCUSSION: The radiographs show a well-defined, irregular, eccentric lesion in the distal tibia metaphysis with a thin sclerotic margin. The radiographs are diagnostic of nonossifying fibroma, a common entity in this age group and in this location. No further work-up is indicated. The patient was asymptomatic prior to the injury and the lesion is small and thus not worrisome for an impending pathologic fracture; therefore, no treatment is indicated beyond observation. The natural history of these lesions is to gradually ossify as the patient reaches skeletal maturity.
REFERENCES: Menendez LR (ed): Orthopaedic Knowledge Update: Musculoskeletal Tumors. Rosemont, IL, American Academy of Orthopaedic Surgeons, 2002, pp 69-75.
Biermann JS: Common benign lesions of bone in children and adolescents. J Pediatr Orthop 2002;22:268-273.
4. A healthy 52-year-old woman is seeking professional advice about management of osteoporosis. She has no risk factors for osteoporosis. What is the best recommendation for bone health for this patient?
1- Bone mineral density testing performed semiannually
2- No treatment
3- A healthy diet high in calcium
4- 1,000 to 1,500 mg calcium supplement plus 400 to 800 IU vitamin D per day
5- Estrogen therapy
PREFERRED RESPONSE: 4
DISCUSSION: Women older than age 50 years should receive daily supplementation with calcium and vitamin D to help preserve bone density. Bone mineral density testing is recommended for women age 65 years or older and postmenopausal women with at least one risk factor for osteoporotic fractures: prior fragility fracture, low estrogen levels, premature menopause, long-term secondary amenorrhea, glucocorticoid therapy, maternal history of hip fracture, or low body mass index. Hormone therapy is not approved for the treatment of osteoporosis.
REFERENCES: Gass M, Dawson-Hughes B: Preventing osteoporosis-related fractures: An overview. Am J Med 2006;119:S3-S11.
Lin JT, Lane JM: Osteoporosis: A review. Clin Orthop Relat Res 2004;425:126-134.
5. A 14-year-old boy reports progressive right wrist pain. Radiographs are shown in
Figure 3a, and a photomicrograph is shown in Figure 3b. What is the most likely diagnosis?
1- Enchondroma
2- Nonossifying fibroma
3- Chondromyxoid fibroma
4- Osteosarcoma
5- Chondrosarcoma
PREFERRED RESPONSE: 3
DISCUSSION: The radiographs show a benign-appearing, well-defined lytic lesion with a thin rim of surrounding reactive bone. The photomicrograph shows spindle cells with a myxoid cartilaginous matrix. These findings are diagnostic of chondromyxoid fibroma. This is a rare, benign tumor that usually causes pain and can be locally aggressive.
REFERENCES: Lersundi A, Mankin HJ, Mourikis A, et al: Chondromyxoid fibroma: A rarely encountered and puzzling tumor. Clin Orthop Relat Res 2005;439:171-175.
Menendez LR (ed): Orthopaedic Knowledge Update: Musculoskeletal Tumors. Rosemont, IL, American Academy of Orthopaedic Surgeons, 2002, pp 103-111.
6. A 37-year-old man pulled his hamstring playing softball 3 weeks ago. The patient had not noted any mass prior to his injury. MRI scans of the posterior thigh are shown in Figures 4a and 4b. Figure 4c shows the biopsy specimen from a needle biopsy. What is the most likely diagnosis?
1- Intramuscular hematoma
2- Lipoma
3- Myositis ossificans
4- Malignant fibrous histiocytoma
5- Liposarcoma
PREFERRED RESPONSE: 4
DISCUSSION: Malignant fibrous histiocytoma (MFH) is the most common soft-tissue sarcoma. MFH typically presents as a large mass, deep to the fascia with heterogeneous signal on MRI. The MRI scans show a heterogeneous lesion in the posterior thigh. There is significant high signal uptake on the T2-weighted image. The histology shows malignant histiocytic cells with marked atypia and pleomorphism. Histology of a hematoma would show only old hemorrhage and some granulation tissue. Lipoma and liposarcoma are both seen as a fat-containing lesion on histology. No significant fat tissue is seen in this histologic specimen. Histology of myositis ossificans would show bone formation.
REFERENCES: Simon SR (ed): Orthopaedic Basic Science. Rosemont, IL, American Academy of Orthopaedic Surgeons, 1994, pp 219-276.
Campanacci M: Bone and Soft Tissue Tumors, ed 2. New York, NY, Springer-Verlag, 1999, pp 965-981.
7. A 16-year-old boy has had left knee pain and swelling after sustaining a minor twisting injury while playing basketball 2 weeks ago. Figures 5a through 5e show the radiograph, MRI scans, and biopsy specimens. What is the most likely diagnosis?
1- Osteomyelitis
2- Tuberculosis
3- Osteosarcoma
4- Ewing’s sarcoma
5- Malignant fibrous histiocytoma (MFH)
PREFERRED RESPONSE: 4
DISCUSSION: The imaging studies and histology are most consistent with Ewing’s sarcoma. Tuberculosis can show small round blue cells on histology (lymphocytes associated with chronic infection) but would more typically involve the knee joint and periarticular bone. Osteosarcoma and MFH do not have small round blue cells histologically.
REFERENCES: Sissons HA, Murray RO, Kemp HBS: Orthopaedic Diagnosis. Berlin, Springer-Verlag, 1984, pp 254-256.
Wafa H, Grimer RJ: Surgical options and outcomes in bone sarcoma. Expert Rev Anticancer Ther 2006;6:239-248.
8. A 45-year-old man has had left thigh pain for the past 4 months. An AP radiograph, bone scan, MRI scans, and biopsy specimens are shown in Figures 6a through 6f. What is the most appropriate treatment?
1- Physical therapy
2- Medical management
3- Radiation therapy
4- Prophylactic internal fixation
5- Wide resection
PREFERRED RESPONSE: 2
DISCUSSION: The radiograph demonstrates thickened trabeculae and thickened cortices in the left proximal femur compared to the right, and the bone scan shows increased uptake in this area. The MRI scans show thickened trabeculae with normal marrow signal. These findings are diagnostic of Paget’s disease. Medical treatment, including bisphosphonates and calcitonin, is indicated for painful bone lesions.
REFERENCES: Hadjipavlou AG, Gaitanis IN, Kontakis GM: Paget’s disease of the bone and its management. J Bone Joint Surg Br 2002;84:160-169.
Vaccaro AR (ed): Orthopaedic Knowledge Update 8. Rosemont, IL, American Academy of Orthopaedic Surgeons, 2005, pp 187-196.
9. A 13-year-old boy has a painless “knot” over his left hip. History reveals that he injured his left hip playing soccer 4 months ago. A radiograph and MRI scan obtained at the time of injury are shown in Figures 7a and 7b. He is very active and is currently asymptomatic. A current radiograph is shown in Figure 7c. What is the next most appropriate step in management?
1- Observation
2- Anti-inflammatory medication
3- Referral to a rheumatologist
4- Biopsy
5- Resection of the lesion
PREFERRED RESPONSE: 1
DISCUSSION: The diagnosis is myositis ossificans resulting from an injury. The initial radiograph reveals a small amount of mineralization in the soft tissues overlying the left hip. The MRI scan shows signal abnormality of the entire gluteus minimus muscle with a mineralized mass in the center. The current radiograph shows a lesion within the abductor musculature with mature ossification peripherally. The imaging studies are diagnostic and the patient is asymptomatic; therefore, the management of choice is observation with no further evaluation or treatment indicated.
REFERENCES: Miller AE, Davis BA, Beckley OA: Bilateral and recurrent myositis ossificans in an athlete: A case report and review of treatment options. Arch Phys Med Rehabil 2006;87:286-290.
Vaccaro AR (ed): Orthopaedic Knowledge Update 8. Rosemont, IL, American Academy of Orthopaedic Surgeons, 2005, pp 15-27.
10. Figure 8a shows the clinical photograph of an 83-year-old woman who has an enlarging left forearm mass. MRI scans are shown in Figures 8b and 8c. What is the next most appropriate step in management?
1- Radiation therapy
2- Needle biopsy
3- Marginal resection
4- Chemotherapy
5- Amputation
PREFERRED RESPONSE: 2
DISCUSSION: Any large (greater than 5 cm), deep, heterogeneous mass in the extremities should be considered a sarcoma until proven otherwise. Sarcomas are rare, and without a high index of suspicion, the lesions may be misdiagnosed or there may be a delay in diagnosis. Needle biopsies can obtain sufficient tissue for diagnosis and are associated with less morbidity than open biopsy. Marginal resections or excisional biopsies should be reserved for a few select benign lesions and locations.
REFERENCES: Damron TA, Beauchamp CP, Rougraff BT, et al: Soft-tissue lumps and bumps. Instr Course Lect 2004;53:625-637.
Sim FH, Frassica FJ, Frassica DA: Soft-tissue tumors: Diagnosis, evaluation, and management. J Am Acad Orthop Surg 1994;2:202-211.
11. An otherwise healthy 33-year-old man who works in construction reports a 3-month history of knee pain. Radiographs are shown in Figures 9a and 9b. An axial T1-weighted MRI scan with contrast, an angiogram, and histologies are shown in Figures 9c through 9f. What is the most likely diagnosis?
1- Conventional osteosarcoma
2- Myositis ossificans
3- Osteochondroma
4- Parosteal osteosarcoma
5- Dedifferentiated parosteal osteosarcoma
PREFERRED RESPONSE: 5
DISCUSSION: Dedifferentiated parosteal osteosarcoma designates high-grade transformation of conventional low-grade parosteal osteosarcoma. Unlike conventional parosteal osteosarcoma, where wide surgical excision alone is considered adequate treatment, patients with dedifferentiated osteosarcoma are treated with neoadjuvant chemotherapy and wide local resection. Recognition of dedifferentiated areas with angiography can localize the area that should be biopsied and thus render an accurate diagnosis. Percutaneous biopsy of hypervascular areas should prompt the administration of chemotherapy and wide local excision to optimize patient outcome.
REFERENCES: Sheth DS, Yasko AW, Raymond AK, et al: Conventional and dedifferentiated parosteal osteosarcoma: Diagnosis, treatment, and outcome. Cancer 1996;78:2136-2145.
Lewis VO, Gebhardt MC, Springfield DS: Parosteal osteosarcoma of the posterior aspect of the distal part of the femur: Oncological and functional results following a new resection technique. J Bone Joint Surg Am 2000;82:1083-1088.
12. A 20-year-old man has a large soft-tissue mass behind his knee. MRI scans are shown in Figures 10a through 10c. Figure 10d shows a clinical photograph of his chest. The patient’s condition is most likely a result of a defect in what gene?
1- NF1
2- EWS
3- EXT1
4- P53
5- Rb
PREFERRED RESPONSE: 1
DISCUSSION: The patient has a plexiform neurofibroma and multiple café-au-lait spots, all characteristic of von Recklinghausen’s neurofibromatosis. This disease has been linked to a defect of the gene NF1 on chromosome 17. EWS is one of the genes associated with the 11;22 translocation found in Ewing’s sarcoma and several other sarcomas. EXT1 is the most common gene affecting patients with multiple hereditary exostosis. P53 and Rb are tumor suppressor genes whose inactivation has been associated with tumors in conditions such as Li-Fraumeni and retinoblastoma, respectively.
REFERENCES: Theos A, Korf BR, American College of Physicians, et al: Pathophysiology of neurofibromatosis Type 1. Ann Intern Med 2006;144:842-849.
Menendez LR: Orthopaedic Knowledge Update: Musculoskeletal Tumors. Rosemont, IL, American Academy of Orthopaedic Surgeons, 2002.
13. A 35-year-old man reports the development of a painful 2-cm nodule on his dorsal wrist over the past 3 years. A surgeon excised the lesion with a presumptive diagnosis of a ganglion cyst. Histology sections from the excision are shown in Figures 11a and 11b. What is the most likely diagnosis?
1- Ganglion cyst
2- Clear cell sarcoma
3- Epithelioid sarcoma
4- Epidermal inclusion cyst
5- Synovial sarcoma
PREFERRED RESPONSE: 2
DISCUSSION: The histologic appearance of the soft-tissue lesion reveals compact nests of cells with a clear cytoplasm surrounded by a delicate border of fibrocollagenous tissue. There can be scattered multinucleated giant cells. This is consistent with a clear cell sarcoma, also called malignant melanoma of soft parts. This tumor is usually positive for S-100 and HMB45 (a melanoma-associated antigen). These tumors are frequently found around the foot and ankle. Similar to epithelioid sarcoma, it is usually intimately bound to tendons or tendon sheaths. Often the tumors are present for many years. The classic histologic appearance of this lesion differentiates it from the other choices.
REFERENCES: Enzinger FM, Weiss SW: Soft Tissue Tumors, ed 3. St Louis, MO, Mosby, 1995, p 913.
Lucas DR, Nascimento AG, Sim FH: Clear cell sarcoma of soft tissues: Mayo Clinic experience with 35 cases. Am J Surg Pathol 1992;16:1197-1204.
14. Figures 12a through 12e show the radiograph, MRI scans, and biopsy specimens of a
17-year-old boy. What is the most likely diagnosis?
1- Giant cell tumor
2- Chondroblastoma
3- Clear cell chondrosarcoma
4- Osteosarcoma
5- Tuberculous septic arthritis
PREFERRED RESPONSE: 2
DISCUSSION: The images show an epiphyseal lesion. The MRI scan shows extensive bone edema surrounding the lesion, consistent with chondroblastoma. Histology shows polygonal chondroblasts in a cobblestone-like pattern and areas of calcification consistent with chondroblastoma. Although some giant cells are seen, the age of the patient and the polygonal chondroblasts differentiate this lesion from giant cell tumor. Clear cell chondrosarcoma is an epiphyseal lesion that occurs in an older population, and the cells have clear cytoplasm. This lesion is not producing bone on imaging or histologic specimen, eliminating osteosarcoma. Tuberculous septic arthritis can be an epiphyseal lesion, but granulomas would be seen on histology.
REFERENCES: Menendez LR (ed): Orthopaedic Knowledge Update: Musculoskeletal Tumors. Rosemont, IL, American Academy of Orthopaedic Surgeons, 2002, pp 103-111.
Campanacci M: Bone and Soft Tissue Tumors, ed 2. New York, NY, Springer-Verlag, 1999,
pp 247-263.
15. An 18-year-old boy reports increasing pain with weight bearing on his right leg and at night. Examination reveals swelling around the right midcalf. Radiographs and an MRI scan are shown in Figures 13a through 13c, and a biopsy specimen is shown in
Figure 13d. What is the preferred treatment?
1- Chemotherapy and surgical resection
2- Debridement and IV antibiotics
3- Chemotherapy alone
4- Radiation therapy alone
5- Surgical resection alone
PREFERRED RESPONSE: 1
DISCUSSION: The findings are consistent with Ewing’s sarcoma. The radiographs reveal a lytic lesion in the diaphysis of the right fibula. There is elevation of the periosteum and evidence of a surrounding soft-tissue mass. The biopsy specimen shows diffuse small round blue cells surrounding the lamellar bone. It is the second most common malignant bone tumor in children. The most common treatment regimen consists of chemotherapy followed by surgical resection and/or radiation therapy. Surgical resection is employed when the lesion can be removed with wide margins and causes less morbidity than radiation therapy.
REFERENCES: McCarthy EF, Frassica FJ: Pathology of Bone and Joint Disorders with Clinical and Radiographic Correlation. Philadelphia, PA, WB Saunders, 1998, p 258.
Gibbs CP Jr, Weber K, Scarborough MT: Malignant bone tumors. Instr Course Lect 2002;51:413-428.
16. A 16-year-old girl injured her hip in a fall. Radiographs are shown in Figures 14a and 14b. She denies any history of pain prior to the fall and is currently asymptomatic. A bone scan, MRI scan, and biopsy specimens are shown in Figures 14c through 14f. What is the most likely diagnosis?
1- Osteosarcoma
2- Fibrous dysplasia
3- Osteoblastoma
4- Ossifying fibroma
5- Osteomyelitis
PREFERRED RESPONSE: 2
DISCUSSION: Although the classic radiographic appearance of fibrous dysplasia is one of a central metaphyseal lesion with ground glass matrix, it is not unusual to see either a more radiodense-appearing lesion or a more peripheral location. The histologic finding of spicules of woven bone without osteoblastic rimming in a bland fibrous background is diagnostic of fibrous dysplasia. The imaging studies could be consistent with low-grade osteosarcoma, osteoblastoma, or osteomyelitis, but all have a very different histologic picture. Observation is indicated in the absence of symptoms, impending fracture, or deformity. Fibrous dysplasia most commonly occurs in the proximal femur.
REFERENCES: Huvos AG: Bone Tumors: Diagnosis, Treatment, and Prognosis. Philadelphia, PA, WB Saunders, 1991, pp 30-43.
DiCaprio MR, Enneking WF: Fibrous dysplasia: Pathophysiology, evaluation, and treatment.
J Bone Joint Surg Am 2005;87:1848-1864.
17. A 54-year-old woman reports worsening pain in her buttock, especially when sitting for long periods of time. She has occasional pain and paresthesias radiating down her posterior leg. She has no significant medical history. MRI scans are shown in Figures 15a and 15b and a biopsy specimen is shown in Figure 15c. What is the most likely diagnosis?
1- Myxoid liposarcoma
2- Myxoma
3- Malignant fibrous histiocytoma
4- Fibromatosis
5- Neurofibroma
PREFERRED RESPONSE: 5
DISCUSSION: The biopsy specimen shows a wavy collagenous matrix with elongated cells; this is most consistent with neurofibroma. The patient has a mass in the region of the sciatic nerve. Imaging characteristics, homogeneous and very low signal on T1-weighted and very high signal on the T2-weighted sequences, are consistent with a myxoid-type lesion. These include myxoma, myxoid sarcomas, and nerve sheath tumors.
REFERENCES: Campanacci M: Bone and Soft Tissue Tumors, ed 2. New York, NY, Springer-Verlag, 1999, pp 1135-1136
Menendez LR: Orthopaedic Knowledge Update: Musculoskeletal Tumors. Rosemont, IL, American Academy of Orthopaedic Surgeons, 2002, p 251.
18. It has been shown that bisphosphonate-based supportive therapy (pamidronate or zoledronate) reduces skeletal events (onset or progression of osteolytic lesions) both in patients with multiple myeloma and in cancer patients with bone metastasis. The use of biphosphonate therapy has been associated with
1- increased medical complications of treatment.
2- osteonecrosis of the jaw.
3- improved long-term survival rates.
4- anorexia.
5- decreased quality of life measures.
PREFERRED RESPONSE: 2
DISCUSSION: The use of bisphosphonates has been recently associated with the development of osteonecrosis of the jaw. Length of exposure seems to be the most important risk factor for this complication. The type of bisphosphonate may play a role and previous dental procedures may be a precipitating factor. Bisphosphonates are a class of therapeutic agents originally designed to treat loss of bone density (ie, alendronate). The primary mechanism of action of these drugs is inhibition of osteoclastic activity, and it has been shown that these drugs are useful in diseases with propensities toward osseous metastases. In particular, they are effective in diseases in which there is clear upregulation of osteoclastic or osteolytic activity, such as breast cancer and multiple myeloma, and have developed into a mainstay of treatment for individuals with these diseases. Although shown to reduce skeletal events, there has been no improvement in patient survival.
REFERENCES: Bamias A, Kastritis E, Bamia C, et al: Osteonecrosis of the jaw in cancer after treatment with bisphosphonates: Incidence and risk factors. J Clin Oncol 2005;23:8580-8587.
Thakkar SG, Isada C, Smith J, et al: Jaw complications associated with bisphosphonate use in patients with plasma cell dyscrasias. Med Oncol 2006;23:51-56.
Van Poznak C: The phenomenon of osteonecrosis of the jaw in patients with metastatic breast cancer. Cancer Invest 2006;24:110-112.
19. A 12-year-old girl has had pain in her right knee for 1 month that started as activity-related and progressed to night pain. Radiographs are shown in Figures 16a and 16b,
and a biopsy specimen is shown in Figure 16c. What is the recommended treatment?
1- Resection of the distal femur and postoperative chemotherapy
2- Preoperative chemotherapy followed by radiation therapy, then resection of the distal femur
3- Preoperative chemotherapy followed by surgical resection of the lesion and postoperative chemotherapy
4- Preoperative chemotherapy followed by radiation therapy, resection of the distal femur, then postoperative chemotherapy
5- Resection of the distal femur followed by radiation therapy
PREFERRED RESPONSE: 3
DISCUSSION: This is a classic appearance for an osteosarcoma. The radiographs reveal a mixed osteolytic and osteoblastic lesion in a skeletally immature patient in the distal right femoral metaphysis. The pain pattern with progressive symptoms leading to the presence of night pain is also typical for this condition. The biopsy specimen reveals pleomorphic cells and the presence of osteoid. The current standard of care in the treatment of osteosarcoma is neoadjuvant chemotherapy followed by surgical resection or amputation followed by additional postoperative chemotherapy. Osteosarcoma is not radiosensitive.
REFERENCES: Wold LE, Adler CP, Sim FH, et al: Atlas of Orthopedic Pathology, ed 2. Philadelphia, PA, WB Saunders, 2003, p 179.
McCarthy EF, Frassica FJ: Pathology of Bone and Joint Disorders with Clinical and Radiographic Correlation. Philadelphia, PA, WB Saunders, 1998, p 205.
20. A 6-year-old boy presents with a mass and a lucent lesion involving the tibial shaft as seen in Figure 17a. The mass is mildly tender to palpation. The bone scan is focally hot in the tibia. Biopsy specimens are shown in Figures 17b and 17c. What is the most likely diagnosis?
1- Giant cell tumor
2- Nonossifying fibroma
3- Fibrous dysplasia
4- Eosinophilic granuloma
5- Osteofibrous dysplasia
PREFERRED RESPONSE: 5
DISCUSSION: Osteofibrous dysplasia frequently presents at a very young age, usually less than 10 years. In most patients, it involves the anterior cortex of the tibial shaft and minor anterior bowing of the tibia is frequently seen. The lesion is unpredictable in nature, but local recurrence is very high in patients who undergo surgery before 15 years of age.
REFERENCES: Campanacci M, Laus M: Osteofibrous dysplasia of the tibia and fibula. J Bone Joint Surg Am 1981;63:367-375.
McCaffery M, Letts M, Carpenter B, et al: Osteofibrous dysplasia: A review of the literature and presentation of an additional 3 cases. Am J Orthop 2003;32:479-486.
21. Figure 18a shows the clinical photograph of a 31-year-old man who has a slowly growing nodule on his right middle finger. It is minimally tender, and there is no erythema on examination. A biopsy specimen is shown in Figure 18b. What is the most likely diagnosis?
1- Clear cell sarcoma
2- Clear cell carcinoma
3- Epidermal inclusion cyst
4- Nora’s tumor (BPOP)
5- Epithelioid sarcoma
PREFERRED RESPONSE: 5
DISCUSSION: Epithelioid sarcoma is the most common soft-tissue sarcoma in the hand and most commonly occurs in young adults. The tumors can be superficial and may become ulcerated. Deeper lesions are often attached to tendons, tendon sheaths, or fascial structures. These are usually minimally symptomatic. The biopsy specimen reveals the typical appearance of a nodular pattern with central necrosis. They can mimic a necrotizing granulomatous process. Usually there are chronic inflammatory cells along the margin of the tumor nodules. This biopsy specimen does not have the clear cells necessary for a clear cell carcinoma or sarcoma. Nora’s tumor is a bizarre parosteal osteochondromatous proliferation (BPOP) first described in 1983 by the pathologist, Nora. The lesion is defined as a reactive heterotopic ossification and is mostly found in the hands or feet of adults in the third decade of life.
REFERENCES: Enzinger FM, Weiss SW: Soft Tissue Tumors, ed 3. St Louis, MO, Mosby, 1995, p 1074.
Halling AC, Wollan PC, Pritchard DJ, et al: Epithelioid sarcoma: A clinicopathologic review of 55 cases. Mayo Clin Proc 1996;71:636-642.
22. A 17-year-old girl who initially presented as a child with multiple skeletal lesions,
café-au-lait spots, and precocious puberty now has bone pain. A recent bone scan
reveals multiple areas of increased scintigraphic uptake, including bilateral proximal femurs. A radiograph is shown in Figure 19. Besides activity modification, what is the next best line of treatment for decreasing her pain?
1- Bisphosphonates
2- Calcitonin
3- Parathyroid hormone
4- Vitamin D and calcium
5- Methotrexate
PREFERRED RESPONSE: 1
DISCUSSION: McCune-Albright syndrome is the combination of polyostotic fibrous dysplasia, café-au-lait lesions, and endocrine dysfunction. The most common endocrine presentation is precocious development of secondary sexual characteristics. Compared with bone lesions in patients without polyostotic disease, the skeletal lesions in patients with the syndrome tend to be larger, more persistent, and associated with more complications. Bisphosphonate therapy has been shown in several studies to decrease the pain associated with the skeletal lesions of fibrous dysplasia.
REFERENCES: DiCaprio MR, Enneking WF: Fibrous dysplasia: Pathophysiology, evaluation and treatment. J Bone Joint Surg Am 2005;87:1848-1864.
Zacharin M, O’Sullivan M: Intravenous pamidronate treatment of polyostotic fibrous dysplasia associated with McCune Albright syndrome. J Pediatr 2000;137:403-409.
23. What are the four most common soft-tissue sarcomas to spread via the lymph node system?
1- Rhabdomyosarcoma, malignant fibrous histiocytoma, epithelioid sarcoma, clear cell sarcoma
2- Malignant fibrous histiocytoma, synovial sarcoma, clear cell sarcoma, epithelioid sarcoma
3- Liposarcoma, rhabdomyosarcoma, synovial sarcoma, clear cell sarcoma
4- Rhabdomyosarcoma, clear cell sarcoma, epithelioid sarcoma, synovial sarcoma
5- Liposarcoma, clear cell sarcoma, rhabdomyosarcoma, epithelioid sarcoma
PREFERRED RESPONSE: 4
DISCUSSION: Soft-tissue sarcomas most frequently metastasize to the lung, but certain histologic types have a predilection for the lymph node system as well. Rhabdomyosarcoma, clear cell sarcoma, epithelioid sarcoma, and synovial sarcoma are four of the most common types to spread in this fashion. Careful evaluation and/or sentinel lymph node biopsy plays a role in disease staging and prognosis.
REFERENCES: Riad S, Griffin AM, Liberman B, et al: Lymph node metastasis in soft-tissue sarcoma in an extremity. Clin Orthop Relat Res 2004;426:129-134.
Blazer DG III, Sabel MS, Sondak VK: Is there a role for sentinel lymph node biopsy in the management of sarcoma? Surg Oncol 2003;12:201-206.
24. Figures 20a and 20b show the AP and lateral radiographs of a 62-year-old man who has had hip pain for the past 3 weeks. Figure 20c shows a CT scan of the abdomen and pelvis. A needle biopsy specimen is shown in Figure 20d. Preoperative management should include which of the following?
1- Lymphocentigraphy
2- Colonoscopy
3- Bronchoscopy
4- Embolization of the femoral lesion
5- Bone marrow aspiration
PREFERRED RESPONSE: 4
DISCUSSION: The histology shows findings consistent with metastatic renal cell carcinoma. Renal cell carcinoma metastases are extremely vascular. Preoperative embolization helps minimize the amount of blood loss during curettage of these lesions.
REFERENCES: Chatziioannou AN, Johnson ME, Pneumaticos SG, et al: Preoperative embolization of bone metastases from renal cell carcinoma. Eur Radiol 2000;10:593-596.
Sun S, Lang EV: Bone metastases from renal cell carcinoma: Preoperative embolization. J Vasc Interv Radiol 1998;9:263-269.
25. A 15-year-old girl reports a 6-month history of activity-related knee pain and swelling.
A radiograph, MRI scan, and biopsy specimen are shown in Figures 21a through 21c. What is the most likely diagnosis?
1- Enchondroma
2- Giant cell tumor
3- Chondroblastoma
4- Osteoblastoma
5- Chondromyxoid fibroma
PREFERRED RESPONSE: 3
DISCUSSION: The epiphyseal location on the radiograph and MRI scan and the histologic findings of polyhedral cells separated by a chondroid matrix with pericellular, lattice-like “chicken wire” calcification all suggest chondroblastoma. Although giant cell tumors of bone typically occupy an epiphyseal location, they are rare in children and when present are often metaphyseal in skeletally immature patients. Enchondromas and osteoblastomas are generally metaphyseal and, along with giant cell tumors, have very different histology than seen here. Chondromyxoid fibromas are typically metaphyseal in location.
REFERENCES: Huvos AG: Bone Tumors: Diagnosis, Treatment, and Prognosis. Philadelphia, PA, WB Saunders, 1991, pp 295-313.
Lin PP, Thenappan A, Deavers MT, et al: Treatment and prognosis of chondroblastoma. Clin Orthop Relat Res 2005;438:103-109.
26. A 58-year-old woman has a fracture through a metacarpal lesion after a motor vehicle accident. She denies any preinjury symptoms and the fracture heals uneventfully. Based on the radiograph and MRI scans shown in Figures 22a through 22c obtained following fracture healing, follow-up management should consist of
1- curettage.
2- radiation therapy.
3- observation.
4- bisphosphonates.
5- ray resection.
PREFERRED RESPONSE: 3
DISCUSSION: Enchondromas are the most common benign skeletal lesions identified in the bones of the hand. Most are incidentally found or initially become clinically evident after a pathologic fracture. If the patient has a fracture, the hand is immobilized until union. If the lesion is large and further pathologic fractures are expected, then an intralesional curettage and grafting procedure may be warranted. In this patient, the lesion has not significantly altered the size, shape, or morphology of the involved metacarpal head and recurrent fracture is unlikely. Observation with follow-up radiographs is considered appropriate management.
REFERENCES: Campanacci M: Bone and Soft Tissue Tumors, ed 2. New York, NY, Springer-Verlag, 1999, pp 213-228.
Marco RA, Gitelis S, Brebach GT, et al: Cartilage tumors: Evaluation and treatment. J Am Acad Orthop Surg 2000;8:292-304.
27. A 14-year-old girl reports bilateral patellofemoral symptoms. Based on the radiograph and MRI scans shown in Figures 23a through 23d, what is the next most appropriate step in management of the lesion?
1- Open biopsy
2- Observation
3- Cortisone injection
4- Oncology referral
5- CT of the chest
PREFERRED RESPONSE: 2
DISCUSSION: A periosteal desmoid lesion is a tumor simulator. It is characterized by a bone irregularity along the posteromedial aspect of the distal femur at the insertion of the adductor magnus or the origin of the gastrocnemius muscle. It most commonly occurs in patients who are age 10 to 15 years. The lesions are asymptomatic, with no palpable mass, pain, or swelling. They are frequently an incidental finding when radiographs are obtained for nonspecific symptoms or trauma about the knee. Following recognition of the characteristic imaging findings, observation is the management of choice.
REFERENCES: Dunham WK, Marcus NW, Enneking WF, et al: Developmental defects of the distal femoral metaphysis. J Bone Joint Surg Am 1980;62:801-806.
Verdonk PC, Verstraete K, Verdonk R: Distal femoral cortical irregularity in a 13-year old boy: A case report. Acta Orthop Belg 2003;69:377-381.
28. A 13-year-old boy has knee pain after sustaining a mild twisting injury while
playing basketball 4 weeks ago. Radiographs and MRI scans are shown in Figures 24a through 24d, and biopsy specimens are shown in Figures 24e and 24f. Treatment should consist of
1- neoadjuvant chemotherapy followed by surgical resection and reconstruction.
2- chemotherapy followed by radiation therapy.
3- IV antibiotics for 4 weeks, followed by oral antibiotics for 4 weeks.
4- surgical resection and reconstruction followed by chemotherapy.
5- radiation therapy alone.
PREFERRED RESPONSE: 1
DISCUSSION: The imaging studies and histology are consistent with high-grade osteosarcoma. The standard treatment for osteosarcoma is neoadjuvant chemotherapy combined with wide surgical resection that can be performed with amputation or limb salvage depending on characteristics unique to each tumor and each patient. In most patients, limb salvage surgery can be performed with reconstruction using allografts and/or megaprostheses. Osteosarcoma is poorly responsive to radiation therapy. Chemotherapy alone, in the absence of appropriate surgery, has not proven effective.
REFERENCES: Simon MA, Springfield DS: Surgery for Bone and Soft-Tissue Tumors. Philadelphia, PA, Lippincott-Raven, 1998, pp 265-274.
Gibbs CP, Weber K, Scarborough MT: Malignant bone tumors. Instr Course Lect 2002;51:413-428.
29. A 64-year-old man has had increasing pain in the left hip for the past 6 months.
A radiograph and MRI scan are shown in Figures 25a and 25b. Biopsy specimens are shown in Figures 25c and 25d. What is the recommended treatment?
1- Chemotherapy and internal hemipelvectomy
2- Chemotherapy and hindquarter amputation
3- Radiation therapy and internal hemipelvectomy
4- Radiation therapy and hindquarter amputation
5- Hindquarter amputation or internal hemipelvectomy
PREFERRED RESPONSE: 5
DISCUSSION: The radiograph shows a lytic lesion in the left periacetabular area consistent with chondrosarcoma. A large soft-tissue mass is present along with extension through the supra-acetabular region and pubic ramus. The histology shows a hypercellular lesion infiltrating through the bony trabeculae with a basophilic cytoplasm. This is classified as a grade 2 chondrosarcoma. The treatment of a pelvic chondrosarcoma is wide resection via either an internal hemipelvectomy or amputation. Chondrosarcoma requires surgical resection for control and does not traditionally respond to chemotherapy or external beam irradiation therapy.
REFERENCES: Pring M, Weber, KL, Unni KK, et al: Chondrosarcoma of the pelvis: A review of sixty-four cases. J Bone Joint Surg 2001;83:1630-1642.
Wold LE, Adler CP, Sim FH, et al: Atlas of Orthopedic Pathology, ed 2. Philadelphia, PA,
WB Saunders, 2003, p 255.
30. The scoring system for impending pathologic fractures devised by Mirels involves assessment of which of the following factors?
1- Lesion location, amount of pain, lesion type, lesion size (lucent/blastic)
2- Patient’s functional status, lesion location, amount of pain, lesion size
3- Lesion type (lucent/blastic), patient’s functional status, lesion location, amount
of pain
4- Lesion size, lesion type (lucent/blastic), lesion location, patient’s functional status
5- Amount of pain, patient’s functional status, lesion type (lucent/blastic), lesion size
PREFERRED RESPONSE: 1
DISCUSSION: The scoring system published by Mirels in 1989 is based on the following characteristics: the location of the lesion, the amount of pain the patient is experiencing, the type of lesion (either lucent, mixed, or blastic), and the lesion size. The tumor is scored from 1 to 3 in each category and a total score is obtained that correlates to fracture risk. Prophylactic fixation is advised for lesions with scores of higher than 8, and consideration for stabilization should be strongly considered for scores of 8. The Mirels scoring system can be useful as an adjunct to clinical decision making.
REFERENCES: Mirels H: Metastatic disease in long bones: A proposed scoring system for diagnosing impending pathologic fractures. 1989. Clin Orthop Relat Res 2003;415:S4-S13.
Damron TA, Morgan H, Prakash D, et al: Critical evaluation of Mirels’ rating system for impending pathologic fractures. Clin Orthop Relat Res 2003;415:S201-S207.
31. Figures 26a and 26b show the radiograph and MRI scan of a 22-year-old man with knee pain. What is the most likely diagnosis?
1- Osteochondroma
2- Osteoblastoma
3- Osteosarcoma
4- Chondrosarcoma
5- Malignant fibrous histiocytoma of bone
PREFERRED RESPONSE: 1
DISCUSSION: The lesion is an osteochondroma. This is demonstrated by a pedunculated bone-forming lesion where the medullary space of the lesion communicates with the medullary space of the host bone. The cortex of the exostosis is in continuity with the cortex of the underlying bone. The MRI scan reveals that there is no significant cartilage cap, alleviating concern for malignant conversion to a chondrosarcoma. Osteoblastoma and osteosarcoma typically have mixed areas of bone formation and bone destruction. Malignant fibrous histiocytoma of bone is usually purely lytic.
REFERENCES: Vaccaro AR (ed): Orthopaedic Knowledge Update 8. Rosemont, IL, American Academy of Orthopaedic Surgeons, 2005, pp 197-215.
Menendez LR (ed): Orthopaedic Knowledge Update: Musculoskeletal Tumors. Rosemont, IL, American Academy of Orthopaedic Surgeons, 2002, pp 103-111.
32. Which of the following malignant tumors most commonly contains soft-tissue calcifications seen on radiographs or CT?
1- Hemangioma
2- Ewing’s sarcoma
3- Clear cell sarcoma
4- Malignant fibrous histiocytoma
5- Synovial sarcoma
PREFERRED RESPONSE: 5
DISCUSSION: Focal calcifications causing small radiopacities are found in 15% to 20% of synovial sarcomas. Their irregular contours differentiate them from the phleboliths found in a benign hemangioma. Ewing’s sarcoma, clear cell sarcoma, and malignant fibrous histiocytoma do not commonly have calcifications within the lesions.
REFERENCES: Enzinger FM, Weiss SW: Soft Tissue Tumors, ed 3. St Louis, MO, Mosby, 1995, p 761.
Bullough PG: Atlas of Orthopedic Pathology with Clinical and Radiologic Correlations, ed 2. New York, NY, Gower, 1992, p 17.23.
33. Which of the following is most associated with local recurrence of the lesion seen in the radiograph and MRI scan shown in Figures 27a and 27b?
1- Effectiveness of chemotherapy
2- Effect of local adjuvant
3- Open physes
4- Presence of giant cells
5- Effectiveness of embolization
PREFERRED RESPONSE: 3
DISCUSSION: The lesion is an aneurysmal bone cyst. These lesions are known to have a local recurrence rate of 5% to 50%. Young age, open physes, stage, and type of surgical removal and resulting margin have all been shown to affect the recurrence rate. Chemotherapy is not used in the treatment of aneurysmal bone cysts.
REFERENCES: Gibbs CP Jr, Hefele MC, Peabody TD, et al: Aneurysmal bone cyst of the extremities: Factors related to local recurrence after curettage with a high-speed burr. J Bone Joint Surg Am 1999;81:1671-1678.
Vergel De Dios AM, Bond JR, Shives TC, et al: Aneurysmal bone cyst: A clinicopathologic study of 238 cases. Cancer 1992;69:2921-2931.
34. A 33-year-old woman reports a mass on the right hand that has been enlarging for 1 year. An intraoperative photograph is shown in Figure 28a, and a biopsy specimen is shown in Figure 28b. What is the most likely diagnosis?
1- Ganglion cyst
2- Abscess
3- Hematoma
4- Giant cell tumor of tendon sheath
5- Synovial sarcoma
PREFERRED RESPONSE: 4
DISCUSSION: Giant cell tumor of the tendon sheath is the most common solid soft-tissue mass in the hand. These tumors are slow-growing and may be present for months or years before coming to medical attention. Patients usually report mechanical difficulties because of the size or position of the tumor. The gross appearance is that of a lobulated mass that may be multicolored; typically yellow, brown, red, and gray. Histologically the lesion consists of multinucleated giant cells, polygonal mononuclear cells, and histiocytes that may contain abundant hemosiderin or lipid.
REFERENCES: Walsh EF, Mechrefe A, Akelman E, et al: Giant cell tumor of tendon sheath. Am J Orthop 2005;34;116-121.
Weiss SW, Goldblum JR (eds): Enzinger and Weiss’s Soft Tissue Tumors, ed 4. St Louis, MO, Mosby, 2001, pp 1038-1047.
35. A 15-year-old girl has had a painful mass on the medial aspect of her left thigh for the past 5 years. The pain is present only when she is performing athletic activities and is completely relieved with rest. A radiograph and MRI scan are shown in Figures 29a and 29b. The patient and her parents would like to have the mass removed. What further diagnostic studies are required prior to considering surgical resection?
1- Bone scan
2- CT
3- Needle biopsy
4- Incisional biopsy
5- No further tests are needed
PREFERRED RESPONSE: 5
DISCUSSION: The radiograph and MRI scan show a pedunculated lesion arising from the medial aspect of the distal femoral metaphysis. The cortex of the lesion is contiguous with the cortex of the underlying normal bone. Similarly, the medullary canal of the lesion is contiguous with that of the normal bone. These findings are diagnostic of osteochondroma. Rarely a secondary chondrosarcoma can arise in a preexisting osteochondroma. This diagnosis is suggested by identifying a cartilage cap that is greater than 1.5-cm thick in a skeletally mature patient. MRI is the best study to rule out a secondary chondrosarcoma. CT also may be used for this purpose but is not indicated in this patient because an MRI has already been obtained. A bone scan is not useful to identify a secondary chondrosarcoma. Similarly, there is no role for biopsy in this patient. No further tests are needed.
REFERENCES: Menendez LR (ed): Orthopaedic Knowledge Update: Musculoskeletal Tumors. Rosemont, IL, American Academy of Orthopaedic Surgeons, 2002, pp 103-111.
Murphey MD, Choi JJ, Kransdorf, MJ, et al: Imaging of osteochondroma: Variants and complications with radiologic-pathologic correlation. Radiographics 2000;20:1407-1434.
36. A 22-year-old man has mild hip pain bilaterally and multiple skeletal lesions. Based on the pelvic radiograph shown in Figure 30, what is the inheritance pattern for his disorder?
1- X-linked
2- Autosomal recessive
3- Autosomal dominant
4- Mitochondral inheritance
5- Germline mutation
PREFERRED RESPONSE: 3
DISCUSSION: Multiple hereditary exostoses (MHE) is an autosomal dominant disorder manifested by multiple osteochondromas and characteristic skeletal involvement. EXT1 on 8q24.1 and EXT2 on 11p13 are the two genes most strongly associated with MHE. Mutations in these genes affect proper development of endochondral bone, such that in all affected individuals exostoses develop adjacent to the growth plates of long bones, and some exhibit additional bone deformities. Defects in the EXT genes result in increased chondrocyte proliferation and delayed hypertrophic differentiation.
REFERENCES: Stieber JR, Dormans JP: Manifestations of hereditary multiple exostoses. J Am Acad Orthop Surg 2005;13:110-120.
Hilton MJ, Gutierrez L, Martinez DA, et al: EXT1 regulates chondrocyte proliferation and differentiation during endochondral bone development. Bone 2005;36:379-386.
37. An 80-year-old woman notes a painless mass posterior to her left knee. MRI scans are shown in Figures 31a and 31b. What is the best course of action?
1- Observation
2- Medical management
3- Needle biopsy
4- Incisional biopsy
5- Resection
PREFERRED RESPONSE: 1
DISCUSSION: The MRI scans show a popliteal cyst (Baker’s cyst) in its most common location. The cyst emerges from the knee joint between the medial head of the gastrocnemius muscle and the tendon of the semimembranosus muscle. These images are diagnostic; therefore, no further work-up is indicated. Since the patient is asymptomatic, no treatment is necessary.
REFERENCES: Dlabach JA: Nontraumatic soft tissue disorders, in Canale ST (ed): Campbell’s Operative Orthopaedics, ed 10. Philidelphia, PA, Mosby, 2003, vol 1, pp 885-969.
Fritschy D, Fasel J, Imbert JC, et al: The popliteal cyst. Knee Surg Sports Traumatol Arthrosc 2006;14:623-628.
38. A 38-year-old man has an enlarging left paraspinal soft-tissue mass. Based on the
MRI scans and biopsy specimens shown in Figures 32a through 32e, what is the most likely diagnosis?
1- Fibromatosis
2- Well-differentiated liposarcoma
3- Synovial sarcoma
4- Schwannoma
5- Malignant fibrous histiocytoma
PREFERRED RESPONSE: 1
DISCUSSION: Fibromatosis is a benign but aggressive fibrous lesion that principally arises from the connective tissue of muscle and the overlying fascia. The peak incidence is between the ages of 25 and 35 years. Most patients have a deep-seated, firm, poorly circumscribed mass that has grown insidiously and causes little or no pain. MRI is helpful in diagnosing the lesion and in assessing the extent of disease prior to surgical intervention. Histologically, the lesion is poorly circumscribed and infiltrates the surrounding tissue. The lesion appears bland with uniform spindle cells separated by abundant collagen, with little or no cell-to-cell contact. Despite its bland microscopic appearance, the tumor frequently behaves in an aggressive manner. These lesions do not metastasize but have a high incidence of recurrence. Treatment options consist
of surgical resection, radiation therapy, chemotherapeutic protocols, hormone modulation, and/or anti-inflammatory medications.
REFERENCES: Weiss SW, Goldblum JR, Enzinger FM: Enzinger and Weiss’s Soft Tissue Tumors, ed 4. Philadelphia, PA, Elsevier, 2001, pp 309-337.
Spear MA, Jennings LC, Mankin HJ, et al: Individualizing management of aggressive fibromatoses. Int J Radiat Oncol Biol Phys 1998;40:637-645.
39. What is the most common malignancy involving the hand?
1- Epithelioid sarcoma
2- Synovial sarcoma
3- Metastatic lung carcinoma
4- Chondrosarcoma
5- Squamous cell carcinoma
PREFERRED RESPONSE: 5
DISCUSSION: Skin cancers far outnumber primary musculoskeletal malignancies of the hand and the most common of these is squamous cell carcinoma. Metatastic lung carcinoma, while classic for the carcinoma that metastasizes to the hand, does so at an extremely low rate.
REFERENCES: Fink JA, Akelman E: Nonmelanotic malignant skin tumors of the hand. Hand Clin 1995;11:255-264.
Fleegler EJ: Skin tumors, in Green DP, Hotchkiss RN, Pederson WC (eds): Green’s Operative Hand Surgery, ed 4. Philadelphia, PA, Churchill Livingstone, 1999, vol 2, pp 2184-2205.
40. A 38-year-old man who is an avid runner reports a several month history of right hip pain. Based on the radiograph and cross-sectional CT scan shown in Figures 33a and 33b, what is the most likely diagnosis for the lesions seen on the femoral neck?
1- Synovial herniation pits
2- Osteoid osteoma
3- Fibrous dysplasia
4- Metastatic bone disease
5- Multiple enchondromas
PREFERRED RESPONSE: 1
DISCUSSION: Synovial herniation pits or Pitt’s pits are tumor simulators and are incidentally identified on radiographs obtained for either pain or trauma. The main diagnostic pitfall with this lesion is mistakenly identifying it as an osteoid osteoma. Accurate diagnosis is achieved by knowledge of the location and the characteristic imaging appearance. These are common lesions in individuals with femoroacetabular impingement.
REFERENCES: Pitt MJ, Graham AR, Shipman JH, et al: Herniation pit of the femoral neck. Am J Roentgenol 1982;138:1115-1121.
Daenen B, Preidler KW, Padmanabhan S, et al: Symptomatic herniation pits of the femoral neck: Anatomic and clinical study. Am J Roentgenol 1997;168:149-153.
41. A 35-year-old man has had progressive right knee pain for the past 2 months. An AP radiograph, bone scan, MRI scan, and photomicrograph are shown in Figures 34a through 34d. What is the most appropriate treatment of this lesion?
1- Observation
2- Extended curettage with adjuvant treatment
3- Wide resection
4- Radiation therapy
5- Multimodal treatment including chemotherapy and surgery
PREFERRED RESPONSE: 2
DISCUSSION: This is a classic case of giant cell tumor of bone. The radiograph and the MRI scan reveal a purely lytic lesion in the medial femoral condyle. The lesion is well-demarcated without a rim of sclerotic bone. It is eccentrically located and abuts the subchondral bone. The lesion demonstrates increased uptake on a technetium TC 99m bone scan. These imaging studies are highly suggestive of giant cell tumor arising in its most common location. The photomicrograph confirms the diagnosis of giant cell tumor. Based on these findings, the most widely accepted treatment is extended curettage plus a local adjuvant such as polymethylmethacrylate bone cement, argon beam coagulation, liquid nitrogen, and/or phenol.
REFERENCES: Lackman RD, Hosalkar HS, Ogilvie CM, et al: Intralesional curettage for grades II and III giant cell tumors of bone. Clin Orthop Relat Res 2005;438:123-127.
Ward WG Sr, Li G III: Customized treatment algorithm for giant cell tumor of bone: Report of a series. Clin Orthop Relat Res 2002;397:259-270.
42. What is the most common bone tumor in the hand?
1- Periosteal chondroma
2- Subungual exostosis
3- Chondrosarcoma
4- Osteoid osteoma
5- Enchondroma
PREFERRED RESPONSE: 5
DISCUSSION: The most common bone tumor in the hand is an enchondroma. Forty-two percent of these lesions occur in the small tubular bones. They frequently present with a fracture in these locations. Fractures are usually treated nonsurgically. Indications for surgery include patients with symptomatic lesions or those who are considered high risk for recurrent fracture. The histologic appearance of an enchondroma in the hand is more cellular than enchondromas found in the long bones.
REFERENCES: Menendez LR (ed): Orthopaedic Knowledge Update: Musculoskeletal Tumors. Rosemont, IL, American Academy of Orthopaedic Surgeons, 2002, p 103.
Kuur E, Hansen SL, Lindequist S: Treatment of solitary enchondromas in fingers. J Hand Surg Br 1989;14:109-112.
43. A 30-year-old woman has had pain in her right leg for the past 6 months. A lytic lesion is noted in the anterior cortex of the midtibia, extending 5 cm in length without a soft-tissue mass. A radiograph and a biopsy specimen are shown in Figures 35a and 35b. What is the preferred treatment?
1- Debridement and IV antibiotics
2- Wide resection of the lesion
3- Chemotherapy alone
4- Observation
5- Amputation
PREFERRED RESPONSE: 2
DISCUSSION: In an adult with an anterior cortical tibial lesion, this is the classic histologic appearance and anatomic location for an adamantinoma. The histology reveals areas of epithelial cells (in a glandular pattern) within a fibrous stroma. The epithelial cells are shown in nests. They would stain positively for keratin. Adamantinoma is a rare malignant bone tumor with a propensity for late metastasis. It has a high incidence of local recurrence unless resected with a wide margin. Chemotherapy and radiation therapy are not helpful in the treatment of this disease. Amputation generally is not necessary because a diaphyseal resection is usually possible.
REFERENCES: McCarthy EF, Frassica FJ: Pathology of Bone and Joint Disorders with Clinical and Radiographic Correlation. Philadelphia, PA, WB Saunders, 1998, p 263.
Moon NF, Mori H: Adamantinoma of the appendicular skeleton: Updated. Clin Orthop Relat Res 1986;204:215-237.
44. A 75-year-old woman has had severe shoulder pain for the past month. Her medical history includes hypertension and a total nephrectomy for renal cell carcinoma 7 years ago. Radiographs and sagittal MRI scans are shown in Figures 36a through 36d. A bone scan reveals this to be an isolated lesion. Biopsy findings are consistent with metastatic renal cell carcinoma. What is the most appropriate treatment for this patient?
1- Prophylactic stabilization with an intramedullary rod
2- Radiation therapy alone
3- Embolization alone
4- Wide resection and prosthetic reconstruction
5- Prophylactic stabilization with a locking plate and polymethylmethacrylate cement
PREFERRED RESPONSE: 4
DISCUSSION: Resection and reconstruction of this very proximal lesion provides the best chance to avoid hardware complications that may be associated with stabilization procedures. Wide resection of isolated renal cell carcinoma metastasis, which presents distant to the nephrectomy, may improve long-term survival.
REFERENCES: Fuchs B, Trousdale RT, Rock MG: Solitary bony metastasis from renal cell carcinoma: Significance of surgical treatment. Clin Orthop Relat Res 2005;431:187-192.
Jung ST, Ghert MA, Harrelson JM, et al: Treatment of osseous metastases in patients with renal cell carcinoma. Clin Orthop Relat Res 2003;409:223-231.
45. A patient undergoes a simple excision of a 3-cm superficial mass in the thigh at another institution. The final pathology reveals a leiomyosarcoma, without reference to the margins. What is the recommendation for definitive treatment?
1- Repeat wide excision of the tumor bed
2- Observation
3- Radiation therapy to the tumor bed only
4- Chemotherapy
5- Radiation therapy and chemotherapy
PREFERRED RESPONSE: 1
DISCUSSION: Treatment of patients with unplanned excision of soft-tissue sarcomas is challenging. If the margins are positive or unclear, the patient is best managed with repeat excision of the tumor bed, and radiation therapy if the repeat excision does not yield wide margins. In patients with no detectable tumor on physical examination or imaging after unplanned excision, some studies have shown that up to 35% of patients will have residual disease and a poorer local recurrence rate (22% versus 7%). Therefore, whenever feasible,
a reexcision of the tumor bed is recommended.
REFERENCE: Noria S, Davis A, Kandel R, et al: Residual disease following unplanned excision of soft-tissue sarcoma of an extremity. J Bone Joint Surg Am 1996;78:650-655.
46. A 14-year-old girl has had progressive heel pain for the past several months. Based on the radiograph, MRI scan, and biopsy specimens shown in Figures 37a through 37d, treatment should include
1- observation.
2- wide resection.
3- radiation therapy.
4- extended curettage.
5- chemotherapy.
PREFERRED RESPONSE: 4
DISCUSSION: An aneurysmal bone cyst is a benign, locally destructive lesion of bone. Most are seen in patients in the second decade of life. The clinical presentation varies, but most patients have pain, tenderness, swelling, and/or pathologic fracture. Radiographs show a radiolucent lesion sometimes with expansile remodeling of the cortex. MRI best detects the commonly seen fluid-fluid levels associated with this lesion. Histologic findings include
blood-filled spaces with bland fibrous connective tissue septa. The stroma has histiocytes, fibroblasts, scattered giant cells, hemosiderin, and occasional inflammatory cells. Treatment of these lesions consists of extended curettage, plus or minus the use of adjuvants (liquid nitrogen, phenol, argon beam coagulation), and finally filling the bone void (allograft or other bone substitute).
REFERENCES: Gibbs CP Jr, Hefele MC, Peabody TD, et al: Aneurysmal bone cyst of the extremities: Factors related to local recurrence after curettage with a high-speed burr. J Bone Joint Surg Am 1999;81:1671-1678.
Ramirez AR, Stanton RP: Aneurysmal bone cyst in 29 children. J Pediatr Orthop 2002;22:533-539.
47. A 14-year-old boy has had knee pain for the past 2 months. He also has a low-grade fever of 101.3 degrees F (38.5 degrees C). Laboratory studies show a WBC count of 12,100/mm3 and an erythrocyte sedimentation rate of 58/h. A biopsy specimen of a lesion in the distal femoral metaphysis is shown in Figure 38. What is the most appropriate treatment?
1- Radiation therapy alone
2- Intralesional steroid injection
3- Chemotherapy followed by surgery or radiation therapy
4- Debridement and antibiotics
5- Debridement alone
PREFERRED RESPONSE: 4
DISCUSSION: The clinical presentation of this patient is consistent with both acute osteomyelitis and Ewing’s sarcoma. Both entities can be noted in the distal femoral metaphysis although Ewing’s sarcoma is classically noted in the diaphysis. The histology reveals a mixed inflammatory cell infiltrate with neutrophils, plasma cells, lymphocytes, and histiocytes. Ewing’s sarcoma would be a uniform population of small round blue cells without an inflammatory component. Eosinophilic granuloma (EG) is characterized by Langerhans histiocytes and eosinophils. Treatment of EG often consists of an intralesional steroid injection. Treatment of acute osteomyelitis includes surgical debridement and antibiotics.
REFERENCES: McCarthy JJ, Dormans JP, Kozin SH, et al: Musculoskeletal infections in children: Basic treatment principles and recent advancements. Instr Course Lect 2005;54:515-528.
Calhoun JH, Manring MM: Adult osteomyelitis. Infect Dis Clin North Am 2005;19:765-786.
48. A 14-year-old girl reports a 3-week history of anterior thigh pain and a palpable mass after sustaining a soccer-related injury. Examination reveals a tender, firm mass in the midportion of the rectus femoris. MRI scans are shown in Figures 39a through 39c. What is the most appropriate management?
1- Incision and drainage of the abscess
2- Nonsteroidal anti-inflammatory drugs, physical therapy, and a repeat MRI scan in 6 to 8 weeks
3- Open biopsy
4- Hematoma evacuation and musculotendinous repair
5- Primary wide resection followed by radiation therapy
PREFERRED RESPONSE: 2
DISCUSSION: The history, examination, and MRI scan findings are consistent with a midsubstance partial rupture of the rectus femoris muscle. This is an injury masquerading as a “pseudo tumor.” The lack of an appreciable mass effect on the T1-weighted MRI scan, the defined fluid signal on the T2-weighted scans, and the lack of significant contrast enhancement after gadolinium are all most consistent with injury rather than a neoplasm. Most of these injuries respond to nonsurgical management; a few will benefit from late debridement and repair if symptoms fail to resolve in 3 to 6 months. The treatment of choice is nonsurgical management with a follow-up MRI scan to verify that the findings are resolving.
REFERENCES: Hughes C IV, Hasselman CT, Best TM, et al: Incomplete, intrasubstance strain injuries of the rectus femoris muscle. Am J Sports Med 1995;23:500-506.
Temple HT, Kuklo TR, Sweet DE, et al: Rectus femoris muscle tear appearing as a pseudotumor. Am J Sports Med 1998;26:544-548.
49. A 40-year-old man reports an enlarging soft-tissue mass in his right shoulder. Based on the MRI scan and biopsy specimens shown in Figures 40a through 40c, what is the most likely diagnosis?
1- Nodular fasciitis
2- Fibrosarcoma
3- Intramuscular hemangioma
4- Schwannoma
5- Rhabdomyoma
PREFERRED RESPONSE: 1
DISCUSSION: Nodular fasciitis is a pseudosarcomatous, self-limiting reactive process composed of fibroblasts and myofibroblasts. Most patients give a history of a rapidly growing mass that has been present for only a few weeks. Many have pain associated with the mass and can recall a specific traumatic event predating the presence of the lesion. It can occur at any age but is most commonly seen in adults who are 20 to 40 years of age. Histologically, the lesion is composed of predominantly plump, immature-appearing fibroblasts that bear a close resemblance to the fibroblasts found in granulation tissue. Characteristically, the fibroblasts
are arranged in short, irregular bundles and fascicles and are adjacent to collagen and reticulin. The lesions can appear to be more myxoid or more fibrotic in nature and this correlates to the duration of symptoms. The lesions with a short duration of symptoms have a more myxoid appearance in contrast to those of longer duration characterized by hyaline fibrosis.
REFERENCES: Weiss SW, Goldblum JR, Enzinger FM: Enzinger and Weiss’s Soft Tissue Tumors, ed 4. Philadelphia, PA, Elsevier, 2001, pp 250-266.
Wang XL, De Schepper AM, Vanhoenacker F, et al: Nodular fasciitis: Correlation of MRI findings and histopathology. Skeletal Radiol 2002;31;155-161.
50. A 28-year-old woman has had pain in her hand and mild swelling of the little finger for the past 2 months. A radiograph is shown in Figure 41a, and the biopsy specimen is shown in Figures 41b and 41c. What is the most likely diagnosis?
1- Osteochondroma
2- Osteomyelitis
3- Ollier’s disease
4- Brown tumor
5- Enchondroma
PREFERRED RESPONSE: 5
DISCUSSION: The radiographic appearance shows a slightly expansile lesion in the proximal phalanx of the fifth digit typical of an enchondroma. There is a stippled appearance within the bone and no evidence of cortical destruction. The biopsy reveals a cartilage lesion with basophilic cytoplasm. There are some hypercellular areas but no evidence of pleomorphism. Enchondromas in the tubular bones of the hand are usually more cellular than their counterparts in the femur and humerus and should not be considered malignant. No other lesions are noted in the radiograph, so a diagnosis of Ollier’s disease cannot be made. An osteochondroma is a benign surface cartilage tumor. Brown tumor and osteomyelitis can be differentiated from enchondroma based on the histology.
REFERENCES: Wold LE, Adler CP, Sim FH, et al: Atlas of Orthopedic Pathology, ed 2. Philadelphia, PA, WB Saunders, 2003, p 225.
McCarthy EF, Frassica FJ: Pathology of Bone and Joint Disorders with Clinical and Radiographic Correlation. Philadelphia, PA, WB Saunders, 1998, p 227.
51. A 7-year-old girl has had a painful forearm for the past 2 months. Examination reveals fullness on the volar aspect of the forearm. Radiographs and an MRI scan are shown in Figures 42a through 42c. Biopsy specimens are shown in Figures 42d and 42e. What is the most likely diagnosis?
1- Synovial sarcoma
2- Liposarcoma
3- Rhabdomyosarcoma
4- Hemangioma
5- Wilm’s tumor
PREFERRED RESPONSE: 4
DISCUSSION: The radiographs reveal phleboliths on the volar side of the forearm consistent with hemangioma. The MRI scan reveals a rather well-circumscribed in size, irregular in shape, intramuscular soft-tissue mass in the volar aspect of the distal right forearm within the flexor group musculature. The mass demonstrates heterogeneous mixed signal intensity in both T1- and T2-weighted sequences with increased signal intensity on the T1, suggesting fat within the tumor, typical of hemangioma. The postgadolinium-enhanced sequences demonstrate heterogeneous enhancement. The MRI findings are consistent with a soft-tissue hemangioma.
REFERENCES: Garzon M: Hemangiomas: Update on classification, clinical presentation and associate anomalies. Cutis 2000;66:325-328.
Kurkcuoglu IC, Eroglu A, Karaoglanoglu N, et al: Soft tissue hemangioma is a common soft tissue neoplasm. Eur J Radiol 2004;49:179-181.
52. Which of the following is an important factor in performing a proper biopsy?
1- Staying carefully in the proper intermuscular planes
2- Placing multiple drains
3- Dissecting and protecting critical neurovascular structures
4- Using longitudinal incisions in the extremity
5- Avoiding the use of a tourniquet
PREFERRED RESPONSE: 4
DISCUSSION: There are a number of important technical details in performing a biopsy. Incisions should always be longitudinal in the extremity. Good hemostasis is important in avoiding contamination from hematoma. The approach should avoid neurovascular structures, and go through a single muscle belly when possible. Although a frozen section should be obtained to ensure adequate viable tissue has been obtained, definitive diagnosis is not necessary at the time of the frozen section.
REFERENCES: Vaccaro AR (ed): Orthopaedic Knowledge Update 8. Rosemont, IL, American Academy of Orthopaedic Surgeons, 2005, pp 197-215.
Menendez LR (ed): Orthopaedic Knowledge Update: Musculoskeletal Tumors. Rosemont, IL, American Academy of Orthopaedic Surgeons, 2002, pp 29-34.
53. A 16-year-old girl has had painless swelling in her posterior left arm for the past
4 months. A radiograph, MRI scans, and an incisional biopsy specimen are shown in Figures 43a through 43d. What is the cytogenetic translocation most commonly associated with this tumor?
1- (X; 18) (p11; q11)
2- (11; 22) (q24; q12)
3- (12; 22) (q13; q12)
4- (2; 13) (q35; q14)
5- (12; 16) (q13; p11)
PREFERRED RESPONSE: 1
DISCUSSION: This is a case of synovial sarcoma. The radiograph shows some soft-tissue swelling in the upper arm. The MRI scans show a lesion that has increased signal on T2-weighted images and low signal on T1-weighted images. There is a suggestion of a large cystic component to this lesion. The pathology shows a biphasic population of cells, a spindle cell component, and an epithelioid component. Up to 20% of synovial cell sarcomas have areas of cyst formation. The most common cytogenetic translocation with synovial cell sarcoma is X; 18. The 11; 22 translocation is most commonly associated with Ewing’s sarcomas; the 12; 22 translocation is most commonly associated with clear cell sarcomas; the 2; 13 translocation is most commonly associated with alveolar rhabdomyosarcomas, and the 12; 16 translocation is most commonly associated with myxoid liposarcomas.
REFERENCES: Kawai A, Woodruff J, Healey JH, et al: SYT-SSX gene fusion as a determinant of morphology and prognosis in synovial sarcoma. New Engl J Med 1998;338:153-160.
Sandberg AA: Cytogenetics and molecular genetics of bone and soft tissue tumors. Am J Med Genet 2002;115:189-193.
54. A 43-year-old woman is referred after excisional biopsy of a cutaneous soft-tissue mass from her left shoulder. Based on the biopsy specimens shown in Figures 44a and 44b, what is the best course of action?
1- Marginal resection
2- Observation
3- Wide tumor bed resection
4- Radiation therapy
5- Chemotherapy
PREFERRED RESPONSE: 3
DISCUSSION: Dermatofibrosarcoma protuberans (DFSP) is a rare superficial sarcoma that is frequently misdiagnosed at presentation. It is frequently excised prior to suspecting that the lesion is a sarcoma and if not appropriately treated with tumor bed resection to obtain wide margins, these lesions have a high incidence of local recurrence. It is recommended that the wide excision include the deep fascia and a 2.5- to 3-cm cuff of normal-appearing skin. Distant disease spread is rare and usually occurs in the face of a multiply recurrent lesion. Despite the apparent gross circumscription of these lesions, the tumor diffusely infiltrates the dermis and subcutaneous tissues. A characteristic histologic finding can be seen in the deep margins of the tumor where it intricately interdigitates with normal fat.
REFERENCES: Lindner NJ, Scarborough MT, Powell GJ, et al: Revision surgery in dermatofibrosarcoma protuberans of the trunk and extremities. Eur J Surg Oncol
1999;25:392-397.
Weiss SW, Goldblum JR, Enzinger FM: Enzinger and Weiss’s Soft Tissue Tumors, ed 4. Philadelphia, PA, Elsevier, 2001, pp 491-505.
55. A 33-year-old man reports an enlarging painful soft-tissue mass in his right forearm.
A radiograph and MRI scans are shown in Figures 45a through 45c. Treatment should consist of
1- core biopsy.
2- wide resection.
3- radiation therapy.
4- marginal resection.
5- incisional biopsy.
PREFERRED RESPONSE: 4
DISCUSSION: An intramuscular lipoma is a benign soft-tissue lesion that can grow and has a small risk of progressing to a liposarcoma. Radiographs usually show a globular radiolucent mass adjacent to higher-density muscle tissue shadows. When the patient has symptoms and reports an increase in size of the mass, the treatment of choice after appropriate radiographic analysis is complete excision of the mass with marginal resection. Sampling error is a problem with fatty lesions and core or incisional biopsies are frequently unnecessary, especially if an MRI scan of the lesion shows signal intensity that matches subcutaneous fat on all sequences.
REFERENCES: Damron TA: What to do with deep lipomatous tumors. Instr Course Lect 2004;53:651-655.
Gaskin CM, Helms CA: Lipomas, lipoma variants, and well-differentiated liposarcomas (atypical lipomas): Results of MRI evaluations of 126 consecutive fatty masses. Am J Roentgenol 2004;182:733-739.
Rozental TD, Khoury LD, Donthineni-Rao R, et al: Atypical lipomatous masses of the extremities: Outcome of surgical treatment. Clin Orthop Relat Res 2002;398:203-211.
56. A 20-year-old woman has had wrist pain for the past 5 months. A radiograph, MRI scans, and biopsy specimen are shown in Figures 46a through 46d. The patient is then treated with intralesional surgery. The patient should be counseled that her risk of developing lung metastasis is approximately what percent?
1- 0%
2- 4%
3- 15%
4- 25%
5- 50%
PREFERRED RESPONSE: 2
DISCUSSION: Giant cell tumor of bone has about a 2% risk of benign pulmonary metastasis in all cases and 6% risk in recurrent cases. The radiograph and MRI scans show a lytic destructive lesion in the distal radius with no matrix mineralization. The lesion extends up to the subchondral bone. In a young woman, the most likely diagnosis is giant cell tumor of bone, which is supported by the pathology results that show monotonous fibrovascular stroma with numerous multinucleated giant cells where the nuclei that make up the giant cells are identical to the nuclei that make up the background stromal cells.
REFERENCES: Athanasian EA, Wold LE, Amadio PC: Giant cell tumors of the bones in the hand. J Hand Surg Am 1997;22:91-98.
Siebenrock KA, Unni KK, Rock MC: Giant-cell tumor of bone metastasizing to the lungs: A long-term follow-up. J Bone Joint Surg Br 1998;80:43-47.
57. What is the most common location for localized pigmented villonodular synovitis (PVNS) to occur?
1- Ankle
2- Anterior knee
3- Posterior knee
4- Hip
5- Elbow
PREFERRED RESPONSE: 2
DISCUSSION: Localized PVNS is a form of the disease in which synovial proliferation is restricted to one area of a joint and causes the formation of a small mass-like lesion. The true incidence of this is unknown but is probably less common than the diffuse form of the disease. PVNS presents as a usually painful discrete mass. The anterior compartment of the knee is the most common location.
REFERENCES: Tyler WK, Vidal AF, Williams RJ, et al: Pigmented villonodular synovitis.
J Am Acad Orthop Surg 2006;14:376-385.
Kim SJ, Shin SJ, Choi NH, et al: Arthroscopic treatment for localized pigmented villonodular synovitis of the knee. Clin Orthop Relat Res 2000;379:224-230.
58. A 45-year-old man reports right shoulder pain with overhead activities only. Figures 47a through 47d show the radiographs, bone scan, and MRI scan of a lesion of the proximal shoulder. What is the most appropriate treatment?
1- Needle biopsy
2- Incisional biopsy
3- Curettage and grafting
4- Observation
5- En bloc resection
PREFERRED RESPONSE: 4
DISCUSSION: The figures show a lesion of the proximal humerus consistent with an enchondroma. The lesion is calcified on the radiographs. There is no cortical destruction, significant endosteal scalloping, or soft-tissue mass. The bone scan shows mild uptake in the area of the proximal humerus, and the T2-weighted MRI scan shows a lesion with high uptake, suggesting a lesion with high water content. A CT scan could also be obtained to rule out bone destruction or periosteal reaction. Pain with overhead activities is likely related to the rotator cuff. A biopsy is unlikely to add information because of inherent difficulties interpreting low-grade cartilaginous lesions. Curettage and grafting and en bloc resection are excessive treatments for a benign lesion that is apparently asymptomatic. Observation with a follow-up radiograph in 3 to 6 months is appropriate.
REFERENCES: Menendez LR (ed): Orthopaedic Knowledge Update: Musculoskeletal Tumors. Rosemont, IL, American Academy of Orthopaedic Surgeons, 2002, pp 103-111.
Vaccaro AR (ed): Orthopaedic Knowledge Update 8. Rosemont, IL, American Academy of Orthopaedic Surgeons, 2005, pp 197-215.
59. What is the second most common primary bone malignancy in children?
1- Rhabdomyosarcoma
2- Osteosarcoma
3- Ewing’s sarcoma
4- Fibrosarcoma
5- Adamantinoma
PREFERRED RESPONSE: 3
DISCUSSION: Ewing’s sarcoma is the second most common bone tumor in children with an incidence of three per one million Caucasian children younger than 21 years of age. Ewing’s sarcoma is rare in African Americans. Osteosarcoma is the most common bone tumor in children. Rhabdomyosarcoma is the most common soft-tissue sarcoma in children. Fibrosarcoma is a rare primary bone tumor most commonly seen in adults. Adamantinoma is a rare primary bone malignancy also most commonly seen in adults in the tibia.
REFERENCES: Menendez LR (ed): Orthopaedic Knowledge Update: Musculoskeletal Tumors. Rosemont, IL, American Academy of Orthopaedic Surgeons, 2002, p 195.
Gibbs CP Jr, Weber K, Scarborough MT: Malignant bone tumors. Instr Course Lect 2002;51:413-428.
60. An 11-year-old boy sustained an injury to his arm in gym class. He denies prior pain in the arm. Radiographs are shown in Figures 48a and 48b. What is the next most appropriate step in the management of this lesion?
1- Open biopsy followed by curettage and bone grafting
2- MRI, whole-body bone scan, CT of the chest, followed by incisional biopsy
3- Allow the fracture to heal with nonsurgical management and serial radiographs
4- Open biopsy followed by wide resection and reconstruction with osteoarticular allograft
5- Open biopsy followed by wide resection and endoprosthetic replacement
PREFERRED RESPONSE: 3
DISCUSSION: This radiolucent lesion with a “fallen leaf sign” is typical for a unicameral bone cyst(UBC). The most appropriate treatment is to allow the fracture to heal with clinical and radiographic observation. Curettage and bone grafting is not the best initial management for UBC. Wide resection is not indicated for UBC. The proximal humerus is the most common site for UBC. While staging studies consisting of MRI, bone scan, and CT of the chest are appropriate for lesions suspected of being malignant, the classical appearance of this UBC is such that this work-up is not necessary initially. Following fracture healing, aspiration and injection of the cyst may be indicated.
REFERENCES: Dormans JP, Pill SG: Fractures through bone cysts: Unicameral bone cysts, aneurysmal bone cysts, fibrous cortical defects, and nonossifying fibromas. Instr Course Lect 2002;51:457-467.
Deyoe L, Woodbury DF: Unicameral bone cyst with fracture. Orthopedics 1985;8:529-531.
61. An 83-year-old woman reports pain in her left middle finger after a minor injury. Laboratory studies show a WBC count of 7,000/mm3, an erythrocyte sedimentation rate of 3 mm/h, a uric acid of 10.4 mg/dL, and a normal serum protein electrophoresis. Radiographs are shown in Figures 49a and 49b. A core biopsy specimen is shown is Figure 49c. In addition to treatment of the finger fracture, treatment should include
1- colchicine and indomethacin
2- radiation therapy to the left hand.
3- systemic chemotherapy.
4- IV antibiotics.
5- through the wrist amputation.
PREFERRED RESPONSE: 1
DISCUSSION: This clinical picture is most consistent with periarticular erosions from gout. The patient has multiple periarticular lytic lesions in the hand. The laboratory studies show an elevated serum uric acid level, and the biopsy specimen demonstrates acute and chronic inflammation with prominent clefts. Therefore, the preferred treatment is systemic control of her gout. Radiation therapy, chemotherapy, and/or amputation should be considered for a malignancy; however, the pathology does not demonstrate any evidence of pleomorphism, high nuclear-to-cytoplasmic ratio, nuclear atypia, or mitotic activity. Antibiotics for an infectious process is a consideration, but the minimal elevation in the WBC count and erythrocyte sedimentation rate does not support an infectious process.
REFERENCES: Wise CM: Crystal-associated arthritis in the elderly. Clin Geriatr Med 2005;21:491-511.
Mudgal CS: Management of tophaceous gout of the distal interphalangeal joint. J Hand Surg Br 2006;31:101-103.
62. A 21-year-old man has had right groin pain for the past year. A radiograph, CT scan, MRI scans, and a biopsy specimen are shown in Figures 50a through 50e. What is the most likely diagnosis?
1- Chondrosarcoma
2- Osteoblastoma
3- Chondroblastoma
4- Giant cell tumor of bone
5- Subchondral cyst
PREFERRED RESPONSE: 1
DISCUSSION: The pathology demonstrates a very cellular chondroid matrix with multinucleated forms, atypia, and myxomatous regions. This is most consistent with a myxoid chondrosarcoma. The radiograph shows a well-circumscribed lesion in the superior and medial aspect of the right acetabulum. The CT and MRI scans confirm these same findings with no evidence of matrix mineralization or significant surrounding edema. Unfortunately, in this location with this appearance, the radiographic differential diagnosis includes all the diagnoses listed.
REFERENCES: Terek RM: Recent advances in the basic science of chondrosarcoma. Orthop Clin North Am 2006;37:9-14.
Donati D, El Ghoneimy A, Bertoni F, et al: Surgical treatment and outcome of conventional pelvic chondrosarcoma. J Bone Joint Surg Br 2005;87:1527-1530.
Pring ME, Weber KL, Unni KK, et al: Chondrosarcoma of the pelvis: A review of sixty-four cases. J Bone Joint Surg Am 2001;83:1630-1642.
63. A healthy 16-year-old boy has had increasing pain in the right knee for the past 3 months. Examination reveals warmth and swelling around the distal femur. Radiographs and an MRI scan are shown in Figures 51a through 51c, and a biopsy specimen is shown in Figure 51d. What is the most likely diagnosis?
1- Ewing’s sarcoma
2- Osteosarcoma
3- Rhabdomyosarcoma
4- Osteomyelitis
5- Malignant fibrous histiocytoma
PREFERRED RESPONSE: 2
DISCUSSION: The radiographs show a bone-producing lesion in the distal femoral metaphysis in this case of classic osteosarcoma presenting in the most common location, the distal femur. The coronal MRI scan reveals a marrow-occupying lesion with extension into the soft tissues. The histology shows osteoid production by pleomorphic cells consistent with an osteosarcoma. Ewing’s sarcoma is a bone tumor characterized by uniform small blue cells on histology. Rhabdomyosarcoma is the most common childhood soft-tissue sarcoma. Osteomyelitis has an inflammatory appearance on histology. Malignant fibrous histiocytoma of bone has a lytic radiographic appearance and a pleomorphic storiform pattern without osteoid on histology.
REFERENCES: Wold LE, Adler CP, Sim FH, et al: Atlas of Orthopedic Pathology, ed 2. Philadelphia, PA, WB Saunders, 2003, p 179.
McCarthy EF, Frassica FJ: Pathology of Bone and Joint Disorders with Clinical and Radiographic Correlation. Philadelphia, PA, WB Saunders, 1998, p 205.
64. A 10-year-old boy has had wrist pain for the past 3 months. He denies any history of trauma. He reports mild tenderness associated with a palpable mass. A radiograph and biopsy specimens are shown in Figures 52a through 52c. What is the most likely diagnosis?
1- Chondromyxoid fibroma
2- Periosteal chondroma
3- Surface chondrosarcoma
4- Enchondroma
5- Chondroblastoma
PREFERRED RESPONSE: 2
DISCUSSION: The radiograph shows a benign-appearing cortically based lesion eroding the underlying cortex, producing a saucer-shaped defect typical of a periosteal chondroma. The histology shows benign-appearing neoplastic cartilage. Although enchondroma would have the same histologic appearance, radiographs generally show a lesion with a central medullary epicenter. The benign-appearing histology does not support chondrosarcoma. Chondromyxoid fibroma will generally show histologic elements of its fibrous and myxoid components. Chondroblastoma typically demonstrates histologic findings of polyhedral cells separated by a chondroid matrix with pericellular, lattice-like “chicken wire” calcification.
REFERENCES: Schajowicz F: Tumors and Tumorlike Lesions of Bone: Pathology, Radiology, and Treatment, ed 2. Berlin, Springer-Verlag, 1994, pp 147-151.
Weiner SD: Enchondroma and chondrosarcoma of bone: Clinical, radiologic, and histologic differentiation. Instr Course Lect 2004;53:645-649.
65. A 29-year-old woman reports shoulder pain after sustaining a minor fall 6 weeks ago. She has a history of celiac sprue. Radiographs of the forearm and shoulder are shown in Figures 53a and 53b. Which of the following serum abnormalities would be expected?
1- Elevated calcium level
2- Elevated PTH level
3- Elevated 1,25(OH)2D
4- Elevated phosphate level
5- Low alkaline phosphatase level
PREFERRED RESPONSE: 2
DISCUSSION: Celiac sprue results in rapid gastrointestinal transit and fatty stools that impair the absorption of calcium and vitamin D and result in nutritional-deficiency osteomalacia with secondary hyperparathyroidism. The radiographs show marked osteopenia with brown tumors. A pathologic fracture is seen in the proximal humerus through a large brown tumor. Serum findings include low or normal calcium, low phosphate, elevated alkaline phosphatase, low 1,25(OH)2D, and increased PTH levels. Secondary hyperparathyroidism is associated with a variety of conditions including malabsorption syndromes.
REFERENCES: Potts JT: Parathyroid hormone: Past and present. J Endocrinol 2005;187:311-325.
Corazza GR, Di Stefano M, Maurino E, et al: Bones in coeliac disease: Diagnosis and treatment. Best Pract Res Clin Gastroenterol 2005;19:453-465.
Mankin HJ, Mankin CJ: Metabolic bone disease: An update. Instr Course Lect 2003;52:769-784.
66. A 73-year-old man stepped off a street curb and felt a crack in his left hip. He is now unable to bear weight. A radiograph is shown in Figure 54a. Biopsy specimens are shown in Figures 54b and 54c. What is the most likely diagnosis?
1- Synovial sarcoma
2- Adamantinoma - metastatic prostate cancer
3- Chondrosarcoma
4- Dedifferentiated chondrosarcoma
5- Periosteal osteosarcoma
PREFERRED RESPONSE: 4
DISCUSSION: The biopsy specimens reveal a high-grade spindle cell lesion adjacent to an area of benign cartilage. This is consistent with a dedifferentiated chondrosarcoma. The radiograph shows a pathologic fracture through a lesion characterized by calcification within the left greater trochanter. Distal to the area of calcification, there is a more osteolytic, destructive appearance. Synovial sarcoma has a biphasic appearance histologically with areas of glandular differentiation that stain positive with keratin. Metastatic prostate cancer, although osteoblastic in appearance, would have a glandular histologic appearance. There is no cartilage in these lesions. Classic low-grade chondrosarcoma does not have an area of high-grade pleomorphic spindle cells within the lesion. A periosteal osteosarcoma is a surface-based lesion with a sunburst radiographic pattern. Although there may be cartilage in the lesion histologically, there are also malignant cells producing osteoid. Dedifferentiated chondrosarcoma is an aggressive, high-grade variant of chondrosarcoma.
REFERENCES: Wold LE, Adler CP, Sim FH, et al: Atlas of Orthopedic Pathology, ed 2. Philadelphia, PA, WB Saunders, 2003, p 269.
Mercuri M, Picci P, Campanacci L, et al: Dedifferentiated chondrosarcoma. Skeletal Radiol 1995;24:409-416.
67. The biopsy specimens seen in Figures 55a and 55b are from a lytic lesion in the sacrum of a 58-year-old man. What is the most likely diagnosis?
1- Chondrosarcoma
2- Metastatic renal cell carcinoma
3- Chondromyxoid fibroma
4- Chordoma
5- Giant cell tumor
PREFERRED RESPONSE: 4
DISCUSSION: The lesion is a chordoma and the other listed choices can be eliminated based on the histology. Many tumors can occur in the sacrum including chordoma, multiple myeloma, giant cell tumor, aneurysmal bone cyst, and metastatic disease. The histology in this patient shows a lobulated lesion on low power with fibrous septae separating the lobules. At higher magnification, the cells have eosinophilic vacuolated cytoplasm and are called physaliferous cells. Chordoma is a low-grade neoplasm that most commonly occurs in the sacrum and rarely in the base of the skull. The diagnosis is often delayed. Chordoma is thought to originate from notochordal remnants. Chordoma typically occurs in the midline and has an associated soft-tissue mass.
REFERENCES: Wold LE, Adler CP, Sim FH, et al: Atlas of Orthopedic Pathology, ed 2. Philadelphia, PA, WB Saunders, 2003, p 372.
Fuchs B, Dickey ID, Yaszemski MJ, et al: Operative management of sacral chordoma. J Bone Joint Surg Am 2005;87:2211-2216.
Fourney DR, Rhines LD, Hentschel SJ, et al: En bloc resection of primary sacral tumors: Classification of surgical approaches and outcome. J Neurosurg Spine 2005;3:111-122.
68. A 65-year-old man has a painful right hip mass that has been growing for several years. A radiograph, CT scan, and photomicrograph are shown in Figures 56a through 56c. What is the most appropriate treatment?
1- Chemotherapy alone
2- Surgery alone
3- Radiation therapy alone
4- Chemotherapy and surgery
5- Radiation therapy and surgery
PREFERRED RESPONSE: 2
DISCUSSION: This is a conventional chondrosarcoma. The radiograph and the CT scan show a lesion arising from the inferior pubic ramus with a large soft-tissue mass. Abundant punctate, stippled, or “popcorn-like” calcification is present. The photomicrograph demonstrates hypercellular cartilage. Surgical resection is the only effective treatment. Whereas chemotherapy might play a role in the treatment of a dedifferentiated chondrosarcoma, it has no role in the treatment of a conventional chondrosarcoma. Chondrosarcomas are relatively radioresistant.
REFERENCES: Donati D, El Ghoneimy A, Bertoni F, et al: Surgical treatment and outcome of conventional pelvic chondrosarcoma. J Bone Joint Surg Br 2005;87:1527-1530.
Lee FY, Mankin HJ, Fondren G, et al: Chondrosarcoma of bone: An assessment of outcome.
J Bone Joint Surg Am 1999;81:326-338.
Pring ME, Weber KL, Unni KK, et al: Chondrosarcoma of the pelvis: A review of sixty-four cases. J Bone Joint Surg Am 2001;83:1630-1642.
69. An 8-year-old boy is diagnosed with acute onset cauda equina syndrome. A radiograph, MRI scans, and a biopsy specimen are shown in Figures 57a through 57d. What is the most appropriate treatment?
1- Radiation therapy
2- Chemotherapy
3- Wide surgical resection
4- Marginal surgical resection
5- Aspiration and steroid injection
PREFERRED RESPONSE: 4
DISCUSSION: The findings are consistent with an aneurysmal bone cyst. The MRI scan demonstrates a lesion involving the posterior elements of the vertebrae with fluid-fluid levels and neural compression. Fibrovascular tissue with multinucleated giant cells surrounding a vascular lake is seen on the histology. The most appropriate treatment is a marginal resection of the involved posterior elements. Although the recurrence rate can be as high as 25% to 30%, wide surgical resection could result in permanent neurologic injury and is not necessary. Aspiration and steroid injection have been advocated but would not relieve the nerve compression in this patient. Radiation therapy and chemotherapy are not indicated.
REFERENCES: Mankin HJ, Hornicek FJ, Ortiz-Cruz E, et al: Aneurysmal bone cyst: A review of 150 patients. J Clin Oncol 2005;23:6756-6762.
Garg S, Mehta S, Dormans JP: Modern surgical treatment of primary aneurysmal bone cyst of the spine in children and adolescents. J Ped Orthop 2005;25:387-392.
70. A 19-year-old man has had pain and swelling in his left forearm for the past 8 months. Laboratory studies show a mildly elevated WBC count and erythrocyte sedimentation rate. Radiographs are shown in Figures 58a and 58b, a CT scan is shown in Figure 58c, and T1- and T2-weighted MRI scans are shown in Figures 58d and 58e, respectively.
A biopsy specimen is shown in Figure 58f. Immunohistochemistry demonstrates that the lesion is negative for leukocyte common antigen (CD34). What is the most common cytogenetic translocation associated with this lesion?
1- (11; 22) (q24; q12)
2- (X; 18) (p11; q11)
3- (12; 22) (q13; q12)
4- (2; 13) (q35; q14)
5- (12; 16) (q13; p11)
PREFERRED RESPONSE: 1
DISCUSSION: The imaging studies show a permeative lytic destructive lesion in the proximal radius with “hair-on-end” periosteal reaction and a large soft-tissue mass most consistent with Ewing’s sarcoma. The pathology reveals monotonous sheets of “round blue” cells. This limits the differential diagnosis to primary lymphoma of bone versus Ewing’s sarcoma. These are best differentiated by immunohistochemistry, cytogenetics, and flow cytometry. Lymphoma of bone is typically CD34 positive and CD99 negative; whereas, the reverse is true of Ewing’s sarcoma, CD34 negative and CD99 positive. The most common cytogenetic translocation with Ewing’s sarcoma is 11; 22; 21; 22 and 7; 22 translocations have also been reported in Ewing’s sarcomas. The X; 18 translocation is most commonly associated with synovial cell sarcomas; the 12; 22 translocation is most commonly associated with clear cell sarcomas; the 2; 13 translocation is most commonly associated with alveolar rhabdomyosarcomas, and the 12; 16 translocation is most commonly associated with myxoid liposarcomas. Flow cytometry is used to characterize the cell types of lymphomas.
REFERENCES: Womer R: The cellular biology of bone tumors. Clin Orthop Relat Res 1991;262:12-21.
Yamaguchi U, Hasegawa T, Morimoto Y, et al: A practical approach to the clinical diagnosis of Ewing’s sarcoma/primitive neuroectodermal tumour and other small round cell tumours sharing EWS rearrangement using new fluorescence in situ hybridisation probes for EWSR1 on formalin fixed, paraffin wax embedded tissue. J Clin Pathol 2005;58:1051-1056.
Lazar A, Abruzzo LV, Pollock RE, et al: Molecular diagnosis of sarcomas: Chromosomal translocations in sarcomas. Arch Path Lab Med 2006;130:1199-1207.
71. A 9-year-old girl reports progressive right knee pain. Radiographs are shown in
Figures 59a and 59b. Work-up reveals no other sites of disease. Low- and high-power photomicrographs are shown in Figures 59c and 59d. What is the most appropriate treatment?
1- Chemotherapy alone
2- Surgery alone
3- Radiation therapy alone
4- Chemotherapy and surgery
5- Radiation therapy and surgery
PREFERRED RESPONSE: 4
DISCUSSION: This classic case of osteosarcoma illustrates the typical radiographic and histologic characteristics of this disease. The radiographs show an aggressive-appearing lesion of the distal femur. The lesion has both lytic and blastic areas. Periosteal reaction is present in the form of a Codman’s triangle. The radiographs are highly suggestive of osteosarcoma. The photomicrographs show malignant spindle cells that produce osteoid, thus confirming the diagnosis of osteosarcoma. Treatment of osteosarcoma is multimodal including multi-agent chemotherapy and surgery (wide resection or amputation).
REFERENCES: Bacci G, Ferrari S, Bertoni F, et al: Long-term outcome for patients with nonmetastatic osteosarcoma of the extremity treated at the Instituto Ortopedico Rizzoli according to the Instituto Ortopedico Rizzoli/osteosarcoma-2 protocol: An updated report. J Clin Oncol 2000;18:4016-4037.
Bielack SS, Kempf-Bielack B, Delling G, et al: Prognostic factors in high-grade osteosarcoma of the extremities or trunk: An analysis of 1,702 patients treated on neoadjuvant cooperative osteosarcoma study group protocols. J Clin Oncol 2002;20:776-790.
Mankin HJ, Hornicek FJ, Rosenberg AE, et al: Survival data for 648 patients with osteosarcoma treated at one institution. Clin Orthop Relat Res 2004;429:286-291.
72. What is the most significant factor affecting long-term survival for a patient with bone sarcoma?
1- Presence of metastatic disease at presentation
2- Grade of tumor
3- Size of tumor
4- Depth of tumor invasion
5- Presence of necrosis
PREFERRED RESPONSE: 1
DISCUSSION: The most significant impact on long-term survival is the presence or absence of identifiable metastatic disease on initial presentation. All of these factors have been shown to be predictive of long-term survival to varying degrees.
REFERENCES: Enneking WF, Spanier SS, Goodman MA: A system for the surgical staging of musculoskeletal sarcoma. Clin Orthop Relat Res 1980;153:106-120.
Gibbs CP Jr, Weber K, Scarborough MT: Malignant bone tumors. Instr Course Lect 2002;51:413-428.
73. A 75-year-old woman notes a slowly enlarging mass in the right anterior thigh. Her medical history is significant only for hypertension. An MRI scan of her thigh is shown in Figures 60a through 60d. Which of the following surgical margins is the most appropriate for removal of this lesion?
1- Radical
2- Wide
3- Marginal
4- Intralesional
5- Contaminated
PREFERRED RESPONSE: 3
DISCUSSION: The patient has a large deep anterior thigh mass that has imaging characteristics of mature fat. Intramuscular lipomas are effectively treated with marginal resections with very low recurrence rates. Large lipomas often have small amounts of intralesional signal changes frequently representing trapped muscle fibers and do not necessitate more extensive margins.
REFERENCES: Gaskin CM, Helms CA: Lipomas, lipoma variants, and well-differentiated liposarcomas (atypical lipomas): Results of MRI evaluations of 126 consecutive fatty masses. Am J Roentgenol 2004;182:733-739.
Rozental TD, Khoury LD, Donthineni-Rao R, et al: Atypical lipomatous masses of the extremities: Outcome of surgical treatment. Clin Orthop Relat Res 2002;398:203-211.
74. Figures 61a and 61b show the CT and MRI scans of a 40-year-old man who has hip pain. He undergoes total hip arthroplasty and curettage and cementation of the lesion as shown in Figure 61c. Histopathologic photomicrographs of the curettage specimen are shown in Figures 61d and 61e. What is the best course of treatment?
1- Observation
2- Revision and removal of any remaining gross disease
3- Hemipelvectomy
4- Exploration and repeat curettage
5- Hip disarticulation
PREFERRED RESPONSE: 3
DISCUSSION: The definitive surgery would be removal of the entire resection bed, and in this case of dedifferentiated chondrosarcoma, a hemipelvectomy was performed. The MRI and CT scans show an aggressive cartilage lesion. The histology, representative of a dedifferentiated chondrosarcoma, shows a bimorphic low-grade cartilage lesion with high-grade spindle cell sarcoma. The cartilage lesion is usually an enchondroma or low-grade chondrosarcoma. The dedifferentiated portion is typically a malignant fibrous histocytoma, osteosarcoma, or fibrosarcoma.
REFERENCES: Weber KL, Pring ME, Sim FH: Treatment and outcome of recurrent pelvic chondrosarcoma. Clin Orthop Relat Res 2002;397:19-28.
Pring ME, Weber KL, Unni KK, et al: Chondrosarcoma of the pelvis: A review of sixty-four cases. J Bone Joint Surg Am 2001;83:1630-1642
75. Compared to postoperative radiation therapy, preoperative radiation therapy has a higher rate of what complication?
1- Fibrosis
2- Lymphedema
3- Wound necrosis
4- Local recurrence
5- Neuropathy
PREFERRED RESPONSE: 3
DISCUSSION: Radiation therapy is commonly used as an adjuvant in the treatment of soft-tissue sarcomas, but a controversy exists whether it should be preoperative or postoperative. Radiation therapy can be given prior to or following resection of the tumor. Postoperative radiation is usually given in a higher dose to a larger treatment field. This commonly results in a higher incidence of fibrosis and lymphedema. There is no statistical difference in local recurrence rate between the two radiation treatment plans. Neuropathy is more commonly a complication of chemotherapy. Preoperative radiation therapy has been shown to have a higher wound complication rate than postoperative radiation.
REFERENCES: Vaccaro AR (ed): Orthopaedic Knowledge Update 8. Rosemont, IL, American Academy of Orthopaedic Surgeons, 2005, pp 197-215.
Davis AM, O’Sullivan B, Turcotte R, et al: Late radiation morbidity following randomization to preoperative versus postoperative radiotherapy in extremity soft tissue sarcoma. Radiother Oncol 2005;75:48-53.
76. A 23-year-old man has had right posterolateral knee pain and occasional lateral calf dysesthesias for the past 8 months. A radiograph, CT scan, MRI scans, and a biopsy specimen are shown in Figures 62a through 62e. What is the most likely diagnosis?
1- Osteoid osteoma
2- Brodie’s abscess
3- Osteoblastoma
4- Chondroblastoma
5- Osteosarcoma
PREFERRED RESPONSE: 1
DISCUSSION: The radiograph shows an eccentric, cortically based lytic lesion in the proximal fibula. The CT and MRI scans confirm that it is well circumscribed and cortically based with significant surrounding edema. The radiographic differential diagnosis would be a Brodie’s abscess or osteoid osteoma. An osteoblastoma would have to be greater than 2 cm in size. A chondroblastoma may also have significant edema around it, but it is an epiphyseal-based lesion, not cortically based. The well-circumscribed nature of the lesion is not consistent with osteosarcoma. The pathology shows a very cellular and vascular stroma with plump, but not atypical osteoblast cells making a matrix of immature woven bone. There are no abundant inflammatory cells or dead bone suggestive of osteomyelitis or a Brodie’s abscess. Therefore, the clinical and histologic picture is most consistent with an osteoid osteoma. Percutaneous radiofrequency ablation, usually with CT guidance, has become the preferred method for treating most cases of osteoid osteoma.
REFERENCES: Rosenthal DI: Radiofrequency treatment. Orthop Clin North Am 2006;37:475-484.
Ghanem I: The management of osteoid osteoma: Updates and controversies. Curr Opin Pediatr 2006;18:36-41.
77. Figures 63a and 63b show the radiographs of an 11-year-old girl who sustained a twisting injury of the knee playing soccer. She is now asymptomatic. What is the appropriate treatment of the lesion?
1- Needle biopsy
2- Incisional biopsy
3- Observation
4- Curettage and grafting
5- Chemotherapy and en bloc resection
PREFERRED RESPONSE: 3
DISCUSSION: This is a nonossifying fibroma of the proximal tibia. The lesion is eccentric, cortically based, with sclerotic margins and no evidence of a soft-tissue mass. Nonossifying fibromas are benign lesions that need no biopsy or surgical treatment when classic findings appear on radiographs. A follow-up radiograph should be performed 2 to 3 months after the initial presentation to ensure that the lesion is not progressive. Surgery is reserved for large lesions with risk of pathologic fracture or for cases where a displaced pathologic fracture has occurred and internal fixation is needed for fracture treatment. Nondisplaced pathologic fractures through nonossifying fibromas are best treated by allowing the fracture to heal and observation of the lesion.
REFERENCES: Vaccaro AR (ed): Orthopaedic Knowledge Update 8. Rosemont, IL, American Academy of Orthopaedic Surgeons, 2005, pp 197-215.
Menendez LR (ed): Orthopaedic Knowledge Update: Musculoskeletal Tumors. Rosemont, IL, American Academy of Orthopaedic Surgeons, 2002, pp 77-85.
78. A 21-year-old man is referred for evaluation of a lesion in the left proximal femur that was discovered when he was undergoing an upper gastrointestinal series. He reports no hip or thigh pain. Radiographs are shown in Figures 64a and 64b. What is the best course of action?
1- Observation
2- Needle biopsy
3- Incisional biopsy
4- Curettage
5- Wide resection
PREFERRED RESPONSE: 1
DISCUSSION: Melorheostosis is a rare disorder characterized by the classic radiographic appearance of flowing hyperostosis in a long bone. The hyperostosis may be on the periosteal or endosteal surface of the bone and frequently gives the appearance of wax falling down the side of a candle. The radiographs are diagnostic; therefore, no further work-up is indicated.
REFERENCES: Menendez LR (ed): Orthopaedic Knowledge Update: Musculoskeletal Tumors. Rosemont, IL, American Academy of Orthopaedic Surgeons, 2002, pp 87-102.
Rozencwaig R, Wilson MR, McFarland GB Jr: Melorheostosis. Am J Orthop 1997;26:83-89.
79. A 17-year-old boy has had elbow pain for the past 6 weeks. A radiograph, MRI scans, and biopsy specimens are shown in Figures 65a through 65e. What is the most likely diagnosis?
1- Osteoid osteoma
2- Osteoblastoma
3- Osteosarcoma
4- Chondroblastoma
5- Giant cell tumor
PREFERRED RESPONSE: 2
DISCUSSION: The findings are consistent with an osteoblastoma. The radiographs show a bone-forming lesion of the distal humerus. The lesion has an osseous component extending out of the native cortex with a thin sclerotic border. The T2-weighted MRI scan shows the lesion extending anteriorly beyond the native cortex. No fluid-fluid levels are seen. Histology shows large osteoblasts producing osteoid and woven bone. The tissue between the spicules of bone and osteoid contains thin fibrous tissue and capillaries. Osteoid osteoma is a smaller lesion usually with sclerotic reactive bone around a small nidus. The histology differentiates osteoblastoma from osteosarcoma because no malignant cells are seen. There is no cartilage production or chondroblasts in the histologic specimen, eliminating chondroblastoma. Giant cell tumors of bone typically occur in a epiphyseal metaphyseal location, most commonly after skeletal maturity, and contain numerus giant cells.
REFERENCES: Menendez LR (ed): Orthopaedic Knowledge Update: Musculoskeletal Tumors. Rosemont, IL, American Academy of Orthopaedic Surgeons, 2002, pp 87-102.
Campanacci M: Bone and Soft Tissue Tumors, ed 2. New York, NY, Springer-Verlag, 1999,
pp 415-433.
80. What is the most common site of metastases from a soft-tissue sarcoma?
1- Liver
2- Bone
3- Lymph nodes
4- Lungs
5- Muscle
PREFERRED RESPONSE: 4
DISCUSSION: The most common site of metastases from a soft-tissue sarcoma is the lungs and occurs in 40% to 60% of patients. The second most common site of metastases in soft-tissue sarcomas is the lymph nodes. Nodal metastases are seen with regularity in synovial sarcoma, epithelioid sarcoma, and rhabdosarcoma. The liver, brain, bone, and muscle are occasional sites of spread, but the occurrence is very rare.
REFERENCES: Simon SR (ed): Orthopaedic Basic Science. Rosemont, IL, American Academy of Orthopaedic Surgeons, 1994, pp 219-276.
Menendez LR (ed): Orthopaedic Knowledge Update: Musculoskeletal Tumors. Rosemont, IL, American Academy of Orthopaedic Surgeons, 2002, pp 255-259.
81. A 45-year-old woman has had right wrist pain for the past 2 months. A radiograph, bone scan, and MRI scan are shown in Figures 66a through 66c, and a photomicrograph is shown in Figure 66d. What is the most likely diagnosis?
1- Osteomyelitis
2- Giant cell tumor
3- Osteosarcoma
4- Chondroblastoma
5- Histiocytosis
PREFERRED RESPONSE: 2
DISCUSSION: The radiograph shows a purely radiolucent lesion without matrix mineralization in the epiphysis of the distal radius. The lesion is “hot” on bone scan, and the MRI scan reveals cortical destruction with a soft-tissue mass. These findings are most consistent with giant cell tumor. The distal radius is a common location for giant cell tumors. The other options would be very uncommon in this location in a 45-year-old patient. The photomicrograph demonstrates multinucleated giant cells in a sea of mononuclear cells. The nuclei of the giant cells and the nuclei of the mononuclear stromal cells are identical. This feature helps distinguish giant cell tumor from other lesions that might contain giant cells.
REFERENCES: Cheng CY, Shih HN, Hsu KY, et al: Treatment of giant cell tumor of the distal radius. Clin Orthop Relat Res 2001;383:221-228.
Unni KK: Dahlin’s Bone Tumors: General Aspects and Data on 11,087 Cases, ed 5. Philadelphia, PA, Lippincott-Raven, 1996, pp 263-284.
82. An otherwise healthy 16-year-old dancer reports a 1-month history of leg pain. AP and lateral radiographs of the distal femur are shown in Figures 67a and 67b. What is the next most appropriate step in management?
1- Immobilization/casting
2- Rest, limited weight bearing, and repeat radiographs in 4 weeks
3- MRI of the femur
4- Biopsy
5- Chemotherapy
PREFERRED RESPONSE: 3
DISCUSSION: The radiographs show a very ill-defined, aggressive, moth-eaten bony destruction involving the distal left femoral diaphysis just above the metaphyseal junction. The differential diagnosis includes Ewing’s sarcoma, osteosarcoma, lymphoma of bone, eosinophilic granuloma, osteomyelitis, and others. MRI would further define the lesion, and soft-tissue and intramedullary extension. There is aggressive periosteal reaction in the posteromedial aspect of the adjacent lesion with some multilayered components in the distal interface of the periosteum. The lateral radiograph shows cortical penetration and irregular periosteal reaction of the posterior margin, suggesting some posterior soft-tissue extension at this site as well.
REFERENCES: Gebhardt MC, Ready JE, Mankin HJ: Tumors about the knee in children. Clin Orthop Relat Res 1990;255:86-110.
Menendez LR (ed): Orthopaedic Knowledge Update: Musculoskeletal Tumors. Rosemont, IL, American Academy of Orthopaedic Surgeons, 2002, pp 175-186.
83. Acral metastases are most commonly seen in what type of carcinoma?
1- Renal cell
2- Lung
3- Thyroid
4- Breast
5- Ovarian
PREFERRED RESPONSE: 2
DISCUSSION: Metastatic lesions to bone are usually located in the axial and proximal appendicular skeleton. Metastases below the elbow and knee are rare, but when they do occur they are most commonly from lung carcinoma.
REFERENCES: Hayden RJ, Sullivan LG, Jebson PJ: The hand in metastatic disease and acral manifestations of paraneoplastic syndromes. Hand Clin 2004;20:335-343.
Menendez LR (ed): Orthopaedic Knowledge Update: Musculoskeletal Tumors. Rosemont, IL, American Academy of Orthopaedic Surgeons, 2002, p 313.
84. A 47-year-old woman has had left medial clavicle pain for the past 6 months. History is remarkable for mediastinal non-Hodgkin’s lymphoma, treated with mantel radiation
22 years ago. A radiograph, CT scan, MRI scan, and a biopsy specimen are shown in Figures 68a through 68d. What is the most likely diagnosis?
1- Radiation-associated sarcoma
2- Radiation-associated osteonecrosis
3- Clavicular osteomyelitis
4- Sternoclavicular septic arthritis
5- Sternoclavicular degenerative joint disease
PREFERRED RESPONSE: 1
DISCUSSION: Radiation-associated sarcomas typically occur at least 5 years following radiation therapy, in the radiation therapy field, and with different histology than the original disease. The radiograph shows a lytic destructive lesion of the medial clavicle. The radiographic differential could include any of the above etiologies. The CT and MRI scans show this same reaction with extension into the adjacent soft tissue and periosteal reaction. These findings eliminate a degenerative process or radiation-induced osteonecrosis but do not distinguish between a neoplastic and infectious process. A PET scan showed marked uptake in the distal clavicle, which is more consistent with a malignant neoplastic process than a reactive process, like that of an infection. These findings, combined with the cellular atypia and bone formation on the biopsy specimen, confirm the diagnosis of radiation-associated sarcoma. In this older patient, radiation-associated sarcoma appears in an unusual location secondary to her previous radiation treatment in that region, which can occur 3 to 50 years after previous radiation therapy.
REFERENCES: Shaheen M, Deheshi BM, Riad S, et al: Prognosis of radiation-induced bone sarcoma is similar to primary osteosarcoma. Clin Orthop Relat Res 2006;450:76-81.
Tabone MD, Terrier P, Pacquement H, et al: Outcome of radiation-related osteosarcoma after treatment of childhood and adolescent cancer: A study of 23 cases. J Clin Oncol 1999;17:2789-2795.
85. In 1980, a 32-year-old woman was found to have right breast mass, and a biopsy revealed adenocarcinoma. She underwent a mastectomy at that time, with no other treatment. Five years later, she noticed a lump in the left breast and underwent a left mastectomy. Seven lymph nodes were positive. In 2006, she now reports hip and thigh pain for the past 3 months. Figures 69a and 69b show AP and lateral radiographs of the femur. A bone scan shows a solitary lesion. Following radiographic staging, what is the next most appropriate step in management?
1- Protected weight bearing and radiation therapy
2- Intramedullary rodding of the femur with curettage and cementation
3- Intramedullary rodding of the femur without curettage and cementation
4- Biopsy of the lesion
5- Resection and reconstruction
PREFERRED RESPONSE: 4
DISCUSSION: Solitary bone lesions require biopsy, for there is the possibility that the lesion may represent a primary bone sarcoma, which will necessitate a different treatment plan. This is especially true in patients with remote histories of cancer. The most likely cause of a lytic bone lesion in a patient older than age 40 years is a metastatic lesion.
REFERENCES: Rougraff BT, Kneisl JS, Simon MA: Skeletal metastases of unknown origin: A prospective study of a diagnostic strategy. J Bone Joint Surg Am 1993;75:1276-1281.
Simon MA, Karluk MB: Skeletal metastases of unknown origin: Diagnostic strategy for orthopedic surgeons. Clin Orthop Relat Res 1982;166:96-103.
86. What is the mechanism of action of bisphosphonates?
1- Directly stimulating osteoblastic new bone formation
2- Increasing gut absorption of calcium
3- Increasing efficiency of 1,25 dihydroxylation of vitamin D in the kidney
4- Inhibiting bone resorption by osteoclasts
5- Decreasing release of parathyroid hormone
PREFERRED RESPONSE: 4
DISCUSSION: Bisphosphonates are stable analogues of pyrophosphate that have a strong affinity for bone hydroxyapatite; these agents inhibit bone resorption by reducing the recruitment and activity of osteoclasts and increasing apoptosis. Bone formed while patients are receiving bisphosphonate treatment is histologically normal. Bisphosphonates have been shown to be effective in decreasing pathologic fractures, bone pain, and the need for radiation therapy in patients with multiple myeloma and metastatic carcinoma to bone. The most effective method of administration is via monthly intravenous infusion. Osteonecrosis of the mandible is sometimes a complication of this treatment.
REFERENCES: Gass M, Dawson-Hughes B: Preventing osteoporosis-related fractures: An overview. Am J Med 2006;119:S3-S11.
Buckwalter JA, Einhorn TA, Simon SR (eds): Orthopaedic Basic Science: Biology and Biomechanics of the Musculoskeletal System, ed 2. Rosemont, IL, American Academy of Orthopaedic Surgeons, 2000, pp 226-227.
87. A 16-year-old girl has had hip pain for 1 year. Approximately 2 months ago she noted the development of a hard mass in the right buttock that has steadily increased in size. She now reports severe pain in the right buttock, with radiation down the leg and numbness involving the right foot and toes. A radiograph is shown in Figure 70a and an axial postcontrast T1-weighted MRI scan is shown in Figure 70b. A biopsy specimen is shown in Figure 70c. The chest CT shows multiple lung metastases. Treatment of this lesion should consist of
1- surgery alone.
2- neoadjuvant chemotherapy and surgical resection.
3- radiation therapy.
4- chemotherapy and radiation therapy.
5- chemotherapy only.
PREFERRED RESPONSE: 4
DISCUSSION: Ewing’s sarcoma is the second most common primary tumor of bone in children. Depending on the site and extent of disease, chemotherapy, radiation therapy, and surgery are all treatment options. In this patient with extensive pelvic and metastatic disease, chemotherapy and radiation therapy offer the best oncologic control while preserving functional outcome.
REFERENCES: Gibbs CP Jr, Weber K, Scarborough MT: Malignant bone tumors. Instr Course Lect 2002;51:413-428.
Thacker MM, Temple HT, Scully SP: Current treatment for Ewing’s sarcoma. Expert Rev Anticancer Ther 2005;5:319-331.
Weber KL: Current concepts in the treatment of Ewing’s sarcoma. Expert Rev Anticancer Ther 2002;2:687-694.
88. A 55-year-old woman has slowly increasing pain at the distal end of her little finger
that is exacerbated by cold temperatures. She denies any history of trauma to her hands and is employed as a school teacher. The histology of the resected specimen is shown in Figure 71. What is the most likely diagnosis?
1- Inclusion cyst
2- Schwannoma
3- Scleroderma
4- Paranychia
5- Glomus tumor
PREFERRED RESPONSE: 5
DISCUSSION: Glomus tumors are rare vascular lesions typically occurring about the nail of the distal phalanx of the hand. The diagnostic “triad” of glomus tumors consists of local pain, sensitivity to cold, and paroxysmal pain. They tend to present with pain as the most typical symptom and this can be exacerbated by changes in temperature that is felt to cause a vascular response within the lesion. The biopsy specimen confirms a glomus tumor showing the typical vascular spaces surrounded by glomus epithelioid glomus cells.
REFERENCES: Zook EG, Brown RE: The perionychium, in Green DP, Hotchkiss RN, Pederson WC (eds): Green’s Operative Hand Surgery, ed 4. Philadelphia, PA, Churchill Livingstone, 1999, vol 2, pp 1353-1380.
McDermott EM, Weiss AP: Glomus tumors. J Hand Surg Am 2006;31:1397-1400.
89. A 69-year-old woman reports a painful clicking in her right shoulder. A soft-tissue mass is palpated at the lower portion of the scapula. Based on the MRI scan and biopsy specimens shown in Figures 72a through 72c, what is the most likely diagnosis?
1- Lymphoma
2- Malignant fibrous histiocytoma
3- Synovial sarcoma
4- Fibromatosis
5- Elastofibroma
PREFERRED RESPONSE: 5
DISCUSSION: Elastofibroma is a rare, benign soft-tissue pseudotumor characteristically located in the subscapular region. Most patients are between 50 and 70 years of age and have pain, decreased shoulder range of motion, or a mass. The lesion usually is best visualized with the arm elevated forward and adducted to displace the scapula laterally and allow the mass to protrude from the chest wall. They are frequently bilateral. Grossly, the mass is ill-defined, oblong or spherical, firm, and ranges in size from 5 cm to 10 cm. These masses appear infiltrative and frequently are mistaken for a sarcomatous lesion. Histologically, the mass is composed of a mixture of intertwining eosinophilic collagen and elastic fibers, and scattered fibroblasts, mucoid material, and fat.
REFERENCES: Vastamaki M: Elastofibroma scapulae. Clin Orthop Relat Res 2001;392:404-408.
Nielsen T, Sneppen O, Mykre-Jensen O, et al: Subscapular elastofribroma: A reactive pseudotumor. J Shoulder Elbow Surg 1996;5:209-213.
Weiss SW, Goldblum JR, Enzinger FM: Enzinger and Weiss’s Soft Tissue Tumors, ed 4. Philadelphia, PA, Elsevier, 2001, pp 286-289.
90. A 40-year-old man has a painless mass around his left ankle. He notes minimal growth over the past year. An MRI scan is shown in Figure 73a, and biopsy specimens are shown in Figures 73b and 73c. What is the most likely diagnosis?
1- Epithelioid sarcoma
2- Clear cell sarcoma
3- Pigmented villonodular synovitis
4- Malignant fibrous histiocytoma
5- Synovial sarcoma
PREFERRED RESPONSE: 5
DISCUSSION: The biopsy specimen is a low-power view of a soft-tissue sarcoma with a biphasic pattern of epithelial cells and fibrous spindle cells that are typical of a synovial sarcoma. A deep, painless soft-tissue mass greater than 5 cm in size is suspicious for a sarcoma. The imaging in this patient is indeterminate and the patient requires a biopsy for an accurate diagnosis. The biopsy reveals a tumor with a biphasic appearance consistent with a synovial sarcoma. These tumors are slow growing, occur primarily in the lower extremities, and are found in a younger demographic population compared to malignant fibrous histiocytoma and liposarcoma. They can occur in a biphasic pattern with clumps of epithelial cells and fibrous spindle cells or in a monophasic pattern. Synovial sarcomas stain positively for keratin. Keratin is positive in nearly all biphasic types and in many tumors of the monophasic fibrous type. Sixty percent of these tumors are found in the lower extremity. The area around the knee is the most common location, followed by the ankle and foot. Epithelioid and clear cell sarcomas are found associated with tendon sheaths of the hand and feet and are generally smaller in size. Epithelioid sarcoma may resemble granulomatous inflammation histologically; clear cell sarcoma is composed of nests of clear cells with occasional multinucleated giant cells. Pigmented villonodular synovitis is found within the synovium and is characterized by a villous histologic appearance with hemosiderin-laden macrophages. Malignant fibrous histiocytoma has a storiform histologic pattern with an abundance of pleomorphic cells.
REFERENCES: Enzinger FM, Weiss SW: Soft Tissue Tumors, ed 3. St Louis, MO, Mosby, 1995, p 757.
Menendez LR (ed): Orthopaedic Knowledge Update: Musculoskeletal Tumors. Rosemont, IL, American Academy of Orthopaedic Surgeons, 2002, p 227.
91. A 28-year-old woman has left shoulder pain and a tender soft-tissue mass. Based on the MRI scan and biopsy specimens shown in Figures 74a through 74c, what is the most likely diagnosis?
1- Malignant fibrous histiocytoma
2- Lymphoma
3- Synovial sarcoma
4- Schwannoma
5- Neurofibroma
PREFERRED RESPONSE: 4
DISCUSSION: Schwannomas (neurilemomas) occur at all ages but are most frequently seen in persons between the ages of 20 and 50 years. MRI features of schwannomas are fairly nonspecific, but when they are associated with a large named nerve, the identification of a mass in continuity with that nerve is highly suggestive of a schwannoma. Most have a fairly homogeneous appearance with a high water content and often fusiform shape. Classically, the histology shows alternating Antoni A (dense spindle cell region) areas and Antoni B (loose myxoid tissue) areas. They also demonstrate uniform intense immunostaining with S-100 protein.
REFERENCES: Damron TA, Sim FH: Soft-tissue tumors about the knee. J Am Acad Orthop Surg 1997;5:141-152.
Weiss SW, Goldblum JR, Enzinger FM: Enzinger and Weiss’s Soft Tissue Tumors, ed 4. Philadelphia, PA, Elsevier, 2001, pp 1146-1167.
92. Which of the following prognostic indicators is associated with the least favorable outcome for patients newly diagnosed with osteosarcoma?
1- High histologic grade
2- Tumor size greater than 8 cm
3- Extracompartmental growth
4- Solitary pulmonary metastasis
5- Solitary bone metastasis
PREFERRED RESPONSE: 5
DISCUSSION: Distant bone metastasis is associated with an extremely poor prognosis for patients with osteosarcoma (5-year survival rate of less than 10%). Most osteosarcomas are high grade and extracompartmental, and approximately half are greater than 8 cm at presentation. The 5-year survival rate for these patients is approximately 70%. Patients with a solitary pulmonary metastasis have a prognosis worse than patients without detectable metastases but not as bad as those with bone metastases.
REFERENCES: Bielack SS, Kempf-Bielack B, Delling G, et al: Prognostic factors in high-grade osteosarcoma of the extremities or trunk: An analysis of 1,702 patients treated on neoadjuvant cooperative osteosarcoma study group protocols. J Clin Oncol 2002;20:776-790.
Heck RK, Stacy GS, Flaherty MJ, et al: A comparison study of staging systems for bone sarcomas. Clin Orthop Relat Res 2003;415:64-71.
Kager L, Zoubeck A, Potschger U, et al: Primary metastatic osteosarcoma: Presentation and outcome of patients treated on neoadjuvant Cooperative Osteosarcoma Study Group protocols.
J Clin Oncol 2003;21:2011-2018.
93. A 12-year-old girl has had increasing left knee pain for the past 3 months. A radiograph is shown in Figure 75a, and low- and high-power photomicrographs are shown in
Figures 75b and 75c. What is the most appropriate treatment?
1- Chemotherapy alone
2- Surgery alone
3- Radiation therapy alone
4- Chemotherapy and surgery
5- Radiation therapy and surgery
PREFERRED RESPONSE: 4
DISCUSSION: The radiograph reveals an aggressive purely lytic lesion of the distal femoral metaphysis. There is no apparent matrix mineralization or periosteal reaction. The photomicrographs show a malignant spindle cell neoplasm in a storiform pattern. Based on these findings, the diagnosis is malignant fibrous histiocytoma of bone. At most institutions, patients with this tumor are treated similar to patients with osteosarcoma with multi-agent chemotherapy and surgery with wide margins (resection or amputation).
REFERENCES: Menendez LR (ed): Orthopaedic Knowledge Update: Musculoskeletal Tumors. Rosemont, IL, American Academy of Orthopaedic Surgeons, 2002, pp 203-209.
Papagelopoulos PJ, Galanis EC, Sim FH, et al: Clinicopathological features, diagnosis, and treatment of malignant fibrous histiocytoma of bone. Orthopedics 2000;23:59-65.
94. A 58-year-old woman has had a slowly progressing mass over the distal interphalangeal (DIP) joint of her dominant hand with a worsening deformity of her nail. She has no significant medical history but underwent bilateral knee arthroplasties 1 year ago. Radiographs reveal a small osteophyte at the DIP joint dorsally. A clinical photograph and a biopsy specimen are shown in Figures 76a and 76b. What is the most likely diagnosis?
1- Metastatic lung carcinoma
2- Mucous cyst
3- Synovial sarcoma
4- Inclusion cyst
5- Felon abscess
PREFERRED RESPONSE: 2
DISCUSSION: A mucous cyst is thought to be a ganglion arising from the DIP joint in patients with osteoarthritis. They are frequently associated with nail deformities. Treatment involves removal of the cyst with debridement of DIP joint osteophytes.
REFERENCES: Fritz GR, Stern PJ, Dickey M: Complications following mucous cyst excision. J Hand Surg Br 1997;22:222-225.
Zook EG, Brown RE: The perionychium, in Green DP, Hotchkiss RN, Pederson WC (eds): Green’s Operative Hand Surgery, ed 4. Philadelphia, PA, Churchill Livingstone, 1999, vol 2, pp 1353-1380.
95. A 13-year-old boy has had pain and swelling in his ankle for the past several months. Based on the radiograph, MRI scan, and biopsy specimen shown in Figures 77a through 77c, what is the best course of action?
1- Observation
2- Curettage and bone grafting
3- Radiofrequency ablation
4- Radiation therapy
5- Wide resection
PREFERRED RESPONSE: 2
DISCUSSION: Chondroblastomas are benign cartilage lesions frequently seen in adolescents or young adults. They are found in the epiphyseal or apophyseal regions of bones. The radiograph shows a radiolucent lesion with mineralization and a well-marginated rim of reactive bone. The lesion is composed of sheets of immature chondroblasts (polygonal cells with a clear, bluish cytoplasm and a small round central nucleus). In some regions, classic “chicken-wire” matrix calcifications and a “cobblestone” pattern of cell arrangement may be seen. Treatment consists of curettage and bone grafting.
REFERENCES: Campanacci M: Bone and Soft Tissue Tumors, ed 2. New York, NY, Springer-Verlag, 1999, pp 247-264.
Lin PP, Thenappan A, Deavers MT, et al: Treatment and prognosis of chondroblastoma. Clin Orthop Relat Res 2005;438:103-109.
96. A 39-year-old man has had a foot mass for the past several months. MRI scans are shown in Figures 78a through 78c. A core biopsy specimen reveals synovial sarcoma, and a staging chest CT scan is normal. Which of the following treatments offers the best local tumor control and expedites the patient’s return to normal function?
1- Transtibial amputation
2- Transmetatarsal amputation
3- Wide resection and radiation therapy
4- Ray resection and radiation therapy
5- Transfemoral amputation
PREFERRED RESPONSE: 1
DISCUSSION: Certain histologic subtypes of soft-tissue sarcoma have been noted to arise preferentially in the hand and the foot, such as epithelioid sarcoma, clear cell sarcoma, and synovial sarcoma. Synovial sarcoma is the most common foot sarcoma. Frequently there is a delay in diagnosis because the lesions are rare. The lesions tend to occur in younger adults, typically between the ages of 15 and 40 years. Patients with hand and foot sarcomas have been described as having improved overall survival, but this is likely a result of the smaller size of tumors arising in these locations. In this patient, the tumor has grown to a substantial size and involves many of the bones of the midfoot. Limb salvage may be a possibility when incorporated into a multidisciplinary treatment program, but this will entail months of adjuvant treatment and significant morbidity. Amputation and early prosthetic fitting still have a role in management of some soft-tissue sarcomas, most frequently in the foot.
REFERENCES: Ferguson PC: Surgical considerations for management of distal extremity soft tissue sarcomas. Curr Opin Oncol 2005;17:366-369.
Scully SP, Temple HT, Harrelson JM: Synovial sarcoma of the foot and ankle. Clin Orthop Relat Res 1999;364:220-226.
97. A 10-month-old boy has multiple skeletal lesions and a skin rash that he has had since he was a newborn. Based on the radiographs and biopsy specimens shown in Figures 79a through 79d, what is the most likely diagnosis?
1- Osteomyelitis
2- Langerhan’s cell histiocytosis
3- Ewing’s sarcoma
4- Lymphoma
5- Neuroblastoma
PREFERRED RESPONSE: 2
DISCUSSION: Langerhans cell histiocytosis or eosinophilic granuloma is a nonneoplastic lesion that is part of a spectrum of clinical diseases featuring histiocytes. Most occur during the first two decades of life within any bone. Radiographs show a radiolucent lesion, frequently diaphyseal in location. A periosteal response is occasionally seen and can resemble more aggressive lesions such as osteomyelitis or Ewing’s sarcoma. Histology demonstrates CD1a positive histiocytes with large oval-shaped nuclei with indentation, and a variable presence of eosinophils.
REFERENCES: Plasschaert F, Craig C, Bell R, et al: Eosinophilic granuloma: A different behaviour in children than in adults. J Bone Joint Surg Br 2002;84:870-872.
Campanacci M: Bone and Soft Tissue Tumors, ed 2. New York, NY, Springer-Verlag, 1999, pp 857-876.
98. Which of the following is not a characteristic of synovial sarcomas?
1- may contain calcifications.
2- occur in younger patients than most soft-tissue sarcomas.
3- are usually intra-articular.
4- often respond well to chemotherapy.
5- often contain a known genetic abnormality.
PREFERRED RESPONSE: 3
DISCUSSION: Synovial sarcomas have a number of features that differentiate them from other soft-tissue sarcomas. They often have small areas of calcifications within the lesion. They occur in a younger patient population than most soft-tissue sarcomas. A subset of patients with synovial sarcoma tend to be chemosensitive. They often contain the SYT-SSX translocation. Although they can occur intra-articular, this is rare, despite their name.
REFERENCES: Campanacci M: Bone and Soft Tissue Tumors, ed 2. New York, NY, Springer-Verlag, 1999, pp 1109-1126.
Simon SR (ed): Orthopaedic Basic Science. Rosemont, IL, American Academy of Orthopaedic Surgeons, 1994, pp 219-276.
99. Mutations of what gene are associated with the subsequent development of osteosarcoma?
1- RB
2- EWS-FLI1
3- EGF-R
4- IGF-R
5- VEGF
PREFERRED RESPONSE: 1
DISCUSSION: Patients with a mutation of the retinoblastoma gene (RB) have an increased likelihood for the development of osteosarcoma. The EWS-FLI1 gene is the fusion product of a chromosomal translocation of 11:22. EGF-R and IGF-R are growth factor receptors, but mutations in these genes have not been correlated with osteosarcoma. VEGF is a proangiogenic molecule that is involved in tumor formation in multiple sites but not the development of osteosarcoma.
REFERENCES: Scholz RB, Kabisch H, Delling G, et al: Homozygous deletion within the retinoblastoma gene in a native osteosarcoma specimen of a patient cured of a retinoblastoma of both eyes. Pediatr Hematol Oncol 1990;7:265-273.
Hansen MF, Koufos A, Gallie BL, et al: Osteosarcoma and retinoblastoma: A shared chromosomal mechanism revealing recessive predisposition. Proc Natl Acad Sci USA 1985;82:6216-6220.
100. A 27-year-old man has had pain in the right index finger for the past 9 months. The pain is completely relieved with ibuprofen. An AP radiograph and CT scan are shown in Figures 80a and 80b. What is the most likely diagnosis?
1- Brodie’s abscess
2- Hyperparathyroidism
3- Stress fracture
4- Enchondroma
5- Osteoid osteoma
PREFERRED RESPONSE: 5
DISCUSSION: Osteoid osteoma is a round or oval, well-circumscribed lesion with a
radiolucent nidus. A small area of calcification may be present within the center of the nidus. The radiolucent nidus is surrounded by a thick rim of sclerotic bone. These diagnostic
features are frequently better seen on CT. An increase in cyclooxygenase activity has been demonstrated within osteoid osteomas, which may explain why aspirin and other nonsteroidal anti-inflammatory drugs classically relieve the pain associated with these lesions.
REFERENCES: Kneisl JS, Simon MA: Medical management compared with operative treatment for osteoid-osteoma. J Bone Joint Surg Am 1992;74:179-185.
Unni KK: Dahlin’s Bone Tumors: General Aspects and Data on 11,087 Cases, ed 5. Philadelphia, PA, Lippincott-Raven, 1996, pp 121-130.
101. An orthopaedic surgeon frequently uses hip and knee prostheses from a specific manufacturer. The surgeon becomes acquainted with the manufacturer’s representative who provides the support for these prostheses in the hospital. They develop a personal relationship outside of work through a common interest in sailing. Together they become interested in buying a sailboat. The manufacture’s representative suggests a partnership in a boat costing $200,000. The manufacture’s representative would purchase a 90% interest and the surgeon a 10% interest in the boat. There would be no restrictions on use of the boat by the surgeon. What should the orthopaedic surgeon do?
1- Accept the offer because it would allow the physician to have a boat and enjoy a hobby of sailing with a minimal financial outlay.
2- Accept this offer because it would have nothing to do with the orthopaedic practice and because the physician feels strongly that it would have no influence on practice decisions.
3- Reject the offer because it represents an unacceptable conflict of interest.
4- Accept the offer but disclose the relationship to patients when using the representative’s products.
5- Accept the offer but only with a equal interest arrangement in the purchase and use of the boat.
PREFERRED RESPONSE: 3
DISCUSSION: Rejecting this proposal is the only appropriate course of action. Accepting it would, in essence, be receiving a huge gift from industry in the form of a sailboat. Physicians frequently assert that they are not influenced by gifts and relationships with industry representatives, but evidence is to the contrary. Such an arrangement constitutes a tremendous incentive to use the manufacturer’s products. The fact that the boat partnership seems completely outside of the orthopaedic business relationship does not excuse it. Conflicts of interest should always be resolved and in the best interest of patient care, and in this case the best course clearly is to avoid the conflict of interest totally. An equal interest in the boat does not eliminate the conflict of interest.
REFERENCES: AAOS Standard of Professionalism -Orthopaedist -Industry Conflict of Interest (Adopted 4/18/07), Mandatory Standard numbers 6-8. http://www3.aaos.org/member/profcomp/SOPConflictsIndustry.pdf
Opinions on Ethics and Professionalism: The Orthopaedic Surgeon’s Relationship with Industry (Document 1204), in Guide to the Ethical Practice of Orthopaedic Surgery, ed 7. Rosemont, IL, American Academy of Orthopaedic Surgeons, 2007, pp 36-40. http://www.aaos.org/about/papers/ethics/1204eth.asp
AdvaMed Code of Ethics on Interactions with Health Care Professionals, Advanced Medical Technology Association, Washington, DC. http://www.AdvaMed.org